Foot and Ankle free MCQS2020 Online
Question 1 of 100
Figures 1 through 3 are the standing radiographs of a 52-year-old woman with symptomatic hallux valgus in the setting of pes planus and hypermobility of the first tarsometatarsal joint. The patient requests corrective surgery for her right foot. What is the most appropriate surgical treatment for her hallux valgus in combination with a second metatarsal osteotomy?
A. Distal osteotomy of the first metatarsal
B. Resection of the base of the proximal phalanx
C. Fusion of the first metatarsal tarsal joint
D. Closing wedge osteotomy of the proximal phalanx
R: C
Fusion of the first metatarsal tarsal joint is a preferred proximal treatment for hallux valgus correction with associated first-ray hypermobility and may require accompanying second metatarsal shortening osteotomy to prevent transfer metatarsalgia. Closing wedge osteotomy of the proximal phalanx osteotomy may be used for the correction of hallux valgus interphalangeus or as an adjunct procedure for hallux valgus surgery. Resection of the base of the proximal phalanx and distal osteotomy of the first metatarsal are both distal procedures, which are not well-suited with hypermobility at the first metatarsal tarsal joint.
Question 2 of 100
What is the most appropriate tendon transfer and augmentation for surgical treatment of a chronic Achilles tendon rupture?
A. Flexor hallucis longus
B. Extensor digitorum longus
C. Tibialis anterior
D. Posterior tibialis
R: A
Flexor hallucis longus tendon transfer provides a biomechanically strong structure to bridge the defect in surgical treatment of a chronic Achilles tendon rupture. Although the flexor hallucis longus remains the optimal tendon for transfer, others have described tendon transfer and augmentation with the plantaris, peroneus brevis, posterior tibial, and flexor digitorum longus. Tibialis anterior and extensor digitorum longus reside in the anterior compartment of the leg and would be out-of-phase transfers.
Question 3 of 100
Figures 1 through 3 are the radiographs of an 18-year-old man who sustained a 4-foot fall from a ladder injuring his wrist and ankle 2 weeks ago. He was seen in the emergency department, diagnosed with a sprain, and was provided a walking boot with crutches. He has been able to weight bear for only very short distances before experiencing significant pain. On examination, there is mild swelling about the ankle and foot and resolving ecchymosis on the medial and lateral hindfoot. Ankle and hindfoot motion are limited because of pain. Tenderness is noted diffusely about the lateral ankle and hindfoot. Motor and sensation appear intact. What is the next step in management of this patient’s injury?
A. CT scan
B. Reassessment in 2 weeks
C. Corticosteroid injection in the ankle
D. Referral to physical therapy for range of motion and peroneal strengthening
R: A
The patient has an occult fracture of the talus. In this patient, a faint fracture line can be seen in the medial talus, best appreciated on the mortise view. This injury is often overlooked and presents as an ankle sprain that fails to get better. Specifically, in this patient, the inability to fully weight bear and ecchymosis should raise concerns for a more severe injury such as a fracture. Many occult fractures do not require surgical intervention, but proper diagnosis is essential for protected weight bearing. The fracture should be better characterized with additional advanced imaging such as a CT or MRI scan. Physical therapy is not indicated at this time. Re-evaluation at a later date only delays care and may risk further displacement of the fracture. Corticosteroid injection is contraindicated in this setting and those of acute trauma.
Question 4 of 100
A 32-year-old woman has had progressive left foot pain over the first metatarsophalangeal (MTP) joint. Footwear is becoming problematic. There is full range of motion of the first MTP with medial eminence pain. Her weight-bearing radiograph reveals a hallux valgus angle (HVA) of 35° and a 1-2 intermetatarsal angle (IMA) of 10° . What is the best next step?
A. Distal metatarsal osteotomy
B. Distal soft-tissue reconstruction
C. Proximal metatarsal osteotomy
D. Lapidus bunionectomy
R: A
Patients with painful progressive hallux valgus are surgical candidates. Presurgical evaluation includes radiographic examination. The IMA between the first and second metatarsals as well as the HVA must be measured. If the IMA is smaller than 15° and the HVA is smaller than 35°, a distal osteotomy is preferred.
Distal soft-tissue reconstruction is only useful for IMAs smaller than 11° and HVAs smaller than 25°. Proximal osteotomies and the Lapidus bunionectomy are reserved for larger hallux valgus deformities with IMAs exceeding 15° and HVAs exceeding 35°.
Question 5 of 100
Figure 1 is the radiograph of a 67-year-old woman with rheumatoid arthritis who reports an 8-month history of increasing pain, swelling, and deformity. Anti-inflammatory drugs, orthotics, and extradepth shoes have failed to provide relief. What is the next most appropriate step in treatment?
A. First metatarsophalangeal joint arthrodesis and lesser metatarsal head resections
B. First metatarsophalangeal joint replacement and lesser metatarsal head resections
C. Keller arthroplasty and lesser metatarsal head resections
D. Distal Chevron osteotomy and lesser metatarsal head resection
R: A
The patient has a severe rheumatoid forefoot deformity involving all metatarsophalangeal joints. Coughlin and Mann have found that 90% of patients have excellent and good results with combined first metatarsophalangeal fusion and lesser metatarsal head resection. Keller arthroplasty does not provide a stable platform for walking and is associated with recurrent deformity and pain. The first metatarsophalangeal joint replacement has not been shown to provide reliable long-term results. Osteotomies may be indicated in patients without erosive joint changes.
Question 6 of 100
A 62-year-old man with diabetes has had 1 week of leg pain. His right lower extremity is red, hot, and swollen. Which physical examination finding helps differentiate Charcot neuro-arthropathy from an acute infection?
A. Elevating the extremity with resolution of the erythema
B. Presence of palpable pulses
C. Insensate to the monofilament
D. Appearance of the contralateral lower extremity
R: A
In a diabetic patient with acute symptoms, Charcot neuroarthropathy needs to be considered. Typically, these patients first present with a red, hot, swollen extremity. On physical examination, raising the affected extremity will improve the redness and swelling in Charcot. Diabetic patients who are insensate to the monofilament are at risk for development of both Charcot and an acute infection.
Question 7 of 100
Figures 1 through 3 are the radiographs of a 60-year-old woman who has pain in the dorsum of her foot with activity. Which type of footwear could be recommended for her as initial nonsurgical management of her pain?
A. Shoes with a one-inch heel lift
B. Rigid rocker bottom shoes
C. Shoes with extra depth toe box
D. Shoes with orthotics with a lateral post
R: B
Midfoot arthritis can cause significant pain and subsequent flatfoot deformity. Patients may also complain of weakness in gait. Initial nonsurgical treatment includes braces, orthotics, and rigid-soled rocker bottom shoes, as well as nonsteroidal anti-inflammatory drugs. If this fails, fusion of the affected joints has been found to be a reliable procedure to improve function and pain relief.
Question 8 of 100
A 38-year-old man with a congenital pes cavus deformity reports lateral foot pain that has become increasingly debilitating. He has calluses over the lateral column and 3/5 muscle strength of the lateral compartment muscles. Nonsurgical management has failed to provide relief. In surgery, he undergoes a plantar fascial release, peroneus longus to brevis transfer, dorsiflexion osteotomy of the first metatarsal, and a Dwyer osteotomy. He has a hyperextended deformity of the first metatarsophalangeal joint. What tendon transfer will help to address this deformity?
A. Flexor hallucis longus
B. Extensor hallucis longus
C. Extensor hallucis brevis
D. Extensor digitorum longus
R: B
In cavus foot reconstructions with a hyperextended deformity of the first metatarsophalangeal joint, a first-toe Jones procedure is indicated. This is an interphalangeal joint fusion of the first toe with an extensor hallucis longus tendon transfer. The flexor hallucis longus, extensor hallucis brevis and extensor digitorum longus tendons are not of adequate length or in the correct direction to correct this deformity. Transfer of the flexor hallucis longus will exacerbate the metatarsophalangeal hyperextension. The extensor hallucis brevis cannot be transferred because it is necessary for active motion of the first metatarsophalangeal joint and to balance the hallux flexors. There is no extensor digitorum longus to the hallux, and it is important to reduce the extension moment on the hallux by transferring the extensor hallucis longus.
Question 9 of 100
Figure 1 is the radiograph of a 45-year-old woman with a moderate bunion deformity. A Chevron osteotomy was performed and after 6 weeks the patient was doing reasonably well. Six months later she reports increasing pain and stiffness in her toe. Clinically the toe is reasonably straight, but she has significant calluses and overload under the second and third metatarsals. A follow-up radiograph is shown in Figure 2. The patient wants to be free of pain. What is the most appropriate treatment?
A. Revision bunion repair with a Lapidus procedure
B. Keller's excision arthroplasty
C. Metatarsophalangeal joint fusion with an autologous bone block
D. Allograft replacement of the metatarsal head
R: C
The patient developed osteonecrosis of the metatarsal head with shortening of the first metatarsal and subsequent overload of the lesser metatarsals. The most reliable option is to perform a metatarsophalangeal joint fusion with an autologous bone block to restore length. A revision bunion repair will not address the arthritic changes. A Keller's excision arthroplasty will further aggravate the lesser metatarsal overload. An allograft replacement of the metatarsal head has a very low predictability rating and is highly experimental.
Question 10 of 100
A 19-year-old man sustained the closed injury shown in Figures 1 and 2 following a fall from a stair. He underwent open reduction and internal fixation (ORIF) without complication. Three months postoperatively, the patient was noted to have increased pain with the radiographs shown in Figures 3 and 4. What is the most appropriate surgical treatment to restore the mortise?
A. Deltoid reconstruction and syndesmotic fusion
B. Deltoid reconstruction, fibular lengthening, revision ORIF syndesmosis
C. Revision syndesmotic fixation with two 4.5 mm quadcortical syndesmotic screws D. Removal of syndesmotic screw with placement of suture button fixation
R: B
At 3 months, the radiographs reveal incongruity of the mortise without radiographic evidence of arthritis. The patient sustained a Weber C fibular fracture with clear disruption of the syndesmosis. The 3-month postoperative radiographs reveal mild shortening of the fibula with lateral subluxation of the talus with failure of the syndesmotic screw. The most effective method at 3 months postoperatively is to attempt salvage of the joint in a young, active patient. The fibular requires lengthening and revision ORIF, deltoid imbrication, and revision stabilization of the syndesmosis with either screw fixation or suture button fixation. The data are clear that the number of screws or the number of cortices (3 vs. 4) does not significantly differ biomechanically. There is recent evidence to suggest that if all parameters are equal, radiographic and clinical outcomes are superior with suture button fixation compared with screw fixation for the syndesmosis. However, no device will substitute for a well-aligned syndesmosis, which is best achieved through direct visualization as opposed to closed reduction using fluoroscopy.
Question 11 of 100
A 43-year-old woman with long-standing rheumatoid arthritis has a large prominence with soft-tissue swelling under the fifth metatarsal head and over the lateral eminence of the fifth metatarsophalangeal (MTP) joint. She has minimal hammer toes with no significant metatarsalgia. Radiographs show a 4-5 intermetatarsal angle of 7° and a congruent fifth MTP joint. What is the recommended surgical treatment to address this problem?
A. Simple exostectomy
B. Metatarsal head excision
C. Diaphyseal metatarsal osteotomy
D. Simple exostectomy with soft-tissue mass excision
R: D
Exostectomy with soft-tissue mass excision is the treatment of choice. The patient has a type 1 bunionette but most of her symptoms are coming from a rheumatoid nodule that is under the metatarsal head. This is mainly a soft-tissue problem and does not require any type of osteotomy because the 4-5 intramedullary angle is minimally elevated. A metatarsal head resection is commonly performed in patients with rheumatoid arthritis, but is not indicated in this patient because there is a normal fifth MTP joint and no metatarsalgia.
Question 12 of 100
Figure 1 is the radiograph of a patient with a history of pain at the insertion of the Achilles tendon who sustained a sudden onset of severe pain when jumping off a horse. On examination, the patient had a loss of plantar flexion with Thompson test and pain with palpation over the distal Achilles. What is the most appropriate treatment?
A. Early rehabilitation in a functional brace with heel lift
B. Direct end-to-end suture repair
C. Casting in equinus
D. Surgical reconstruction with possible flexor hallucis longus (FHL) transfer
R: D
Achilles tendon sleeve avulsions are noted for the small fragments of bone visible on radiographs. These injuries do not have sufficient distal tendon for a direct repair nor is there sufficient bone for internal fixation. The recommended treatment is direct repair of the Achilles tendon to the calcaneus, either with suture anchors or sutures through bone tunnels. It has been proposed that these injuries are associated with insertional disease and that treatment should address any preexisting conditions. Often times, with insertional ruptures, after debridement of the diseased tissue, an FHL transfer may be necessary.
Question 13 of 100
A 45-year-old man has had 6 weeks of pain, worse with exercise, in the distal Achilles tendon proximal to the insertion. What kind of physical therapy is most effective in treating this issue?
A. Concentric strengthening
B. Eccentric strengthening
C. Hydrotherapy
D. Electric stimulation
R: B
Eccentric strengthening has been shown in multiple studies to be more effective than other nonsurgical management options for midsubstance Achilles tendinopathy. Eccentric therapy involves loading the ankle into dorsiflexion, thought to stimulate increased type 1 collagen synthesis, which helps repair the damaged tendon.
Question 14 of 100
A 28-year-old man has a 3-year history of right ankle pain. He reports having an injury 3 years ago and undergoing subsequent arthroscopy and microfracture at that time. He has continued to have pain, locking, and swelling with activity. What factors shown on the MRI scans in Figures 1 through 4 are predictive of a worse clinical outcome?
A. Large size and uncontained lesion
B. Subchondral cysts and uncontained lesion
C. Subchondral cysts and lateral location
D. Location of the legion in the lateral talus
R: A
The most common first-line surgical treatment of osteochondral lesions of the talus is marrow stimulation with either drilling or microfracture. A retrospective study was undertaken to determine which factors are predictive of a poor clinical outcome for patients undergoing surgical treatment of osteochondral defects of the talus with drilling or microfracture. It was found that size >1.5cm2, younger age (<33 years), and uncontained lesions were the factors predictive of a poor clinical outcome. The size of the lesion was found to be the most significant factor in predicting poor clinical outcome and may be a reason to consider a more extensive initial surgical treatment.
Question 15 of 100
Figures 1 and 2 are the AP and lateral radiographs of a 6-year-old boy who has had 2 weeks of intermittent right foot pain that does not interfere with most activities. On examination, the patient has pain and tenderness to palpation over the dorsomedial aspect of the foot. No significant deformities of the foot are noted. What is the best next step?
A. Vascularized bone grafting of navicular
B. Open reduction and internal fixation of the navicular
C. Non-weight-bearing cast for 6 weeks
D. Activity modification with soft arch supports
R: D
Kohler's disease is a rare, self-limiting, osteonecrosis of the navicular bone. It affects boys more than girls and typical age of onset is between 4 and 5 years. The etiology is thought to be related to the mechanical compression of the navicular from the ossified talus and cuneiforms because the navicular is the last tarsal bone to ossify. This mechanical effect compresses the navicular bone's perichondral ring of blood vessels, resulting in ischemia of the central spongy bone and osteonecrosis. The prognosis remains excellent because of this radial arrangement of blood supply. Radiologic findings show patchy areas of navicular with sclerosis, with loss of normal trabecular pattern. Occasionally, the navicular may appear collapsed or may simply have increased density with minimal fragmentation. Management includes pain control and use of soft arch supports or a medial heel wedge. Patients with disabling symptoms may benefit from a short-leg walking cast for 4 to 6 weeks. Symptoms in untreated patients persist longer than symptoms in treated patients. Because this is a self-limited process, treatment does not affect the course of the disease. Patients with persistent pain should be examined for other conditions such as talar coalition.
Question 16 of 100
A 60-year-old active man has lateral ankle pain and subjective instability. Physical examination demonstrates tenderness over the peroneal tendons and lateral ankle ligaments. There is pain with resisted eversion and associated weakness, but no tendon subluxation. Anterior drawer and talar tilt examination demonstrate lateral ligamentous laxity. Standing alignment demonstrates a varus hindfoot and pes cavus. A Coleman block test demonstrates fixed hindfoot varus. Initial nonsurgical management with bracing and physical therapy is unsuccessful. Regarding the surgical management of this patient’s condition, in addition to addressing the patient’s peroneus longus tear and lateral ankle instability, what other procedure is critical for successful treatment?
A. Flexor hallucis longus tendon transfer
B. Ankle arthroscopy
C. Lateralizing calcaneal osteotomy
D. Dorsiflexion osteotomy of first metatarsal
R: C
Peroneal tendon tears and chronic lateral ankle instability are common in the setting of symptomatic cavovarus foot deformity. Chronic varus stress on the ankle causes failure of the lateral supporting structures, resulting in lateral instability and overload of the peroneal tendons, causing tendinopathy and/or rupture. Many peroneal tears are asymptomatic; however, if the tear progresses, it can result in eversion weakness and further stress on the adjacent peroneal tendon and lateral ankle ligaments. In this case, the patient has a chronic tear of the peroneus longus tendon. Failure to address the underlying hindfoot varus deformity through a lateralizing calcaneal osteotomy in this patient may result in failure of the peroneal tendon repair and lateral ligamentous reconstruction. Flexor hallucis longus transfer is indicated as an augmentation if both peroneus longus and brevis are irreparable. Dorsiflexion osteotomy of the first metatarsal alone would not be indicated because the deformity is not flexible based on Coleman block testing. Lastly, in the absence of intra-articular pathology of the ankle, arthroscopy is not indicated.
Question 17 of 100
What is the most common pathogen for soft-tissue infection of the foot caused by a puncture wound?
A. Staphylococcus aureus
B. Pseudomonas aeruginosa
C. Eikenella corrodens
D. Pasteurella multocida
R: A
Staphylococcus and Streptococcus species are the most common causes of soft-tissue infections in the foot due to punctures. Pseudomonas is the most common cause of osteomyelitis of the foot due to puncture wounds. Pasteurella and Eikenella are seen in animal and human bites, respectively.
Question 18 of 100
Figure 1 is the radiograph of a 20-year-old man with chronic medial-sided left midfoot pain stemming from mixed martial arts injury approximately 2 years ago. What is the most appropriate treatment?
A. Open reduction internal fixation
B. Arthrodesis
C. Osteotomy
D. Walking boot
R: B
Arthrodesis is the preferred method of surgical intervention in the setting of chronic Lisfranc injuries and variants with posttraumatic arthrosis. ORIF is appropriate for acute trauma management, osteotomy is indicated for associated foot deformity, and walking boot may provide temporary pain amelioration.
Question 19 of 100
Video 1 is the presurgical lateral ankle examination of a 45-year-old woman who has had pain and discomfort for 2 years along the posterolateral ankle following a sudden dorsiflexion injury. She notes occasional clicking and popping, and she has not experienced resolution of her symptoms despite immobilization and physical therapy. Examination reveals a stable ankle-to-anterior drawer and inversion stress testing. No strength deficit is noted, but she has apprehension with resisted eversion. MR images do not reveal evidence of tendonosis or tear. The most appropriate surgical intervention is
A. imbrication of the lateral collateral ligaments with reinforcement with the extensor retinaculum (modified Brostrom procedure).
B. peroneal tendon synovectomy and tubularization of the peroneus brevis.
C. groove deepening of the fibula with imbrication of the peroneal retinaculum.
D. excision of the peroneus brevis with tenodesis of the proximal stump to the peroneus longus.
R: C
This patient has a clear history of dorsiflexion injury complicated by chronic peroneal tendon dislocation. The symptoms and findings are consistent with dislocation in this particular case. Groove deepening of the posterior fibula with associated imbrication of the peroneal retinaculum is the most effective surgical procedure. Associated synovitis or tendonosis should be addressed. However, failure to deepen the groove and imbricate the retinaculum will result in continued discomfort. Consequently, both responses that involve isolated tendon surgery are not appropriate. Associated subjective instability can be noted in these patients. The examination is critical to determine the stability of the lateral collateral complex, which is intact in this case (so imbrication is not indicated). A sense of apprehension is a common examination finding because patients sense that the peroneals will subluxate with resisted eversion. Placement of the examiner's hands on the peroneals to stabilize the tendons should relieve this apprehension. A patient may not be able to voluntarily dislocate the tendon. Dynamic ultrasound is the most sensitive radiographic examination for detection of dislocation. Intrasheath peroneal subluxation may also occur and is treated similarly.
Question 20 of 100
Figures 1 and 2 are the radiographs of a 52-year-old laborer who has a 2-year history of pain in his great toe while working. He is specifically interested in discussing whether orthotics could be of benefit. Which orthotic is most appropriate for treating this condition?
A. Full-length orthotic with medial posting
B. Full-length orthotic with lateral posting
C. Three-quarter length orthotic with a metatarsal pad
D. Carbon fiber insert with a Morton's extension
R: D
The radiographs show a patient with significant hallux rigidus. Nonsurgical treatment consists of nonsteroidal anti-inflammatory drugs, activity modification, shoewear modification, corticosteroid injections, and orthotics. The classic orthotic used in the treatment of hallux rigidus is a Morton's extension to limit dorsiflexion of the hallux metatarsophalangeal joint.
Question 21 of 100
Figures 1 and 2 are the clinical photographs of a 70-year-old diabetic patient with recurrent callusing and activity-related pain over the tip of his third digit. In addition to pain, he had localized cellulitis that was treated with a short course of oral antibiotics 4 months ago. He has partial improvement with extra-depth shoes, padding, and recessed orthotics. He takes insulin (HbA1c level, 7.2) and underwent a partial hallux amputation for infection 1 year ago. Radiographs are unremarkable. Pulses are present. The skin is intact. His toe deformity is passively correctable. Ankle range of motion shows 15° of dorsiflexion. What is the best next step?
A. Partial digital amputation through the middle phalanx
B. Achilles tendon lengthening
C. Percutaneous flexor tenotomy
D. Clawtoe correction with proximal interphalangeal joint fusion
R: C
This patient has a clawtoe deformity in the setting of diabetes and previous partial hallux amputation. Despite appropriate shoe modifications, he has continued pain, and the toe is at risk for ulceration and infection. Amputation would be an appropriate choice only in the setting of active infection or ulceration. The patient does not have an Achilles contracture; therefore, a lengthening procedure would also not be indicated. A clawtoe deformity correction is reserved for rigid deformities. In addition, this option has more surgical risks in the setting relative to a flexor tenotomy. The flexor tenotomy can be used to improve the deformity and subsequently decrease the contact pressure in this area.
Question 22 of 100
A 29-year-old man is playing basketball when he steps on an opposing player’s foot, sustaining an inversion injury. He has pain on the lateral aspect of his ankle. He is able to weight bear. On physical examination, he is tender over the lateral ankle ligaments. He has no tenderness over the distal fibula. What is the most appropriate initial management of this injury?
A. MRI of the ankle
B. Non-weightbearing short leg cast
C. Ankle brace, weight bearing as tolerated, and range of motion
D. Surgical repair of the lateral ankle ligaments
R: C
This patient has sustained a lateral ligament sprain. This involves the anterior talofibular ligament, calcaneofibular ligament, and possibly the posterior talofibular ligament. The diagnosis often made on physical examination. Radiographs are taken when patients are unable to bear weight, or have tenderness directly over the fibula. Functional rehabilitation has been shown to be more effective than immobilization in the short-term. An MRI is not indictated unless the patient has symptoms that fail to improve over several months. Surgery would be reserved for chronic instability, or a displaced, intra-articular fragment.
Question 23 of 100
Figure 1 is the radiograph of a 48-year-old man. He is of normal height and weight, medically healthy, and in good physical condition. What is the best treatment option?
A. Short-leg non-weight-bearing cast in plantar flexion
B. Excision of the fragment and reattachment of the Achilles tendon into the calcaneus
C. Immediate open reduction and internal fixation
D. Percutaneous reduction and Kirschner wire fixation
R: C
Immediate open reduction and internal fixation of this fracture is required to prevent necrosis of the overlying soft tissue. Because of the power and proximal pull of the triceps surae, nonsurgical management is not indicated with avulsion fractures of the calcaneus. It leaves a large void that will not fill in with bone, leaves the Achilles tendon weak, and has a high complication rate, especially skin breakdown. The Achilles tendon is securely attached to the fractured tuberosity. Bone-to-bone healing is more reliable than detaching the Achilles tendon from the tuberosity and reattaching it to the remainder of the calcaneus. Because of the size of the avulsed fragment, it will be difficult to correctly tension the tendon if the fractured piece is excised. Percutaneous Kirschner wire fixation is not strong enough to provide a stable fixation of the tuberosity, especially in view of the power of the Achilles tendon contracture.
Question 24 of 100
An 18-year-old competitive volleyball player sustains an inversion injury to his ankle 5 weeks prior. During play, he landed on the ankle and felt a pop. He was unable to return and was given an air stirrup and crutches by his trainer. He is able to weight bear but continues to have pain about the lateral ankle and hindfoot with any impact activities to the extent that he can no longer play. He is otherwise healthy and denies any previous difficulty or injury to the foot or ankle. Radiographs and an MRI scan were ordered by his primary care physician and are shown in Figures 1 through 4. Based on these findings, what is the diagnosis?
A. Occult distal tibia fracture
B. Ankle sprain with a torn anterior talofibular ligament (ATFL) and anterior impingement
C. Posterior ankle impingement/os trigonum
D. Coalition of the subtalar joint
R: D
The case is a common presenting scenario for hindfoot coalition, a sprain that fails to get better. Often, a patient will have no preceding symptoms. Sometimes, only a detailed physical examination comparing range of motion with the contralateral side will demonstrate a deficit. The mechanism of injury and location of the pain are not consistent with a symptomatic os trigonum. While there is bony edema in the talus and calcaneus, there is no edema in the posterior distal tibia that might suggest posterior impingement. The ATFL appears intact on the axial images. A fracture is not seen in the distal tibia, although the growth plate can still be visualized on the sagittal MRI scan. The coronal MRI scan shows a fibrous coalition of the middle facet of the subtalar joint. Pain that originates in the subtalar joint may present medially or laterally. Initial management should consist of a period of immobilization, rather than aggressive early functional rehabilitation.
Question 25 of 100
Figures 1 and 2 are the radiographs of an obese 75-year-old man with a rigid acquired flatfoot deformity. What is the best treatment option?
A. Medial calcaneal osteotomy and tendon transfer
B. Lateral column lengthening and tendon transfer
C. Subtalar arthrodesis
D. Triple arthrodesis
R: D
For stage III adult-acquired flatfoot deformity characterized by dysfunction of the posterior tibial tendon, rigid valgus deformity of the hindfoot, and arthritic changes of the hindfoot joints, arthrodesis is the favored procedure. In an overweight patient with degenerative changes affecting the subtalar and Chopart joints, triple arthrodesis is the best treatment option. Subtalar arthrodesis only addresses the talocalcaneal joint and continues to render the patient symptomatic in the talonavicular and calcaneocuboid joints. Advanced stage III disease precludes reconstructive procedures involving calcaneal osteotomy and tendon transfer.
Question 26 of 100
In discussion of surgical treatment of Lisfranc injuries, one potential benefit of primary arthrodesis over open reduction and internal fixation (ORIF) is
A. decreased need for future secondary surgeries.
B. better ability to correct the initial deformity.
C. less risk of shortening and resultant metatarsalgia.
D. lower risk of future adjacent joint arthritis.
R: A
In the treatment of unstable Lisfranc injuries, both arthrodesis and ORIF are viable options. Some studies show better function in patients undergoing fusion versus arthrodesis, while other studies have shown similar functional results. All studies, however, show decreased rates of secondary procedures when undergoing a fusion instead of ORIF. This is due to planned or unplanned hardware removal, and in some who ultimately require fusion for persistent pain after ORIF.
Question 27 of 100
Figures 1 through 4 are the radiographs of a 17-year-old boy who sustains an injury to his left foot while jumping on a trampoline. He undergoes surgical treatment of the injury. What should he be told about the outcome from his injury?
A. Will be able to return to sports
B. Will develop a foot deformity
C. Most patients have no long-term issues from the injury D. Will eventually need an arthrodesis
R: A
The patient sustained a ligamentous Lisfranc injury. Recent studies have shown that recreational athletes treated with open reduction and internal fixation are able to return to some form of sport. Ligamentous Lisfranc injuries do have a risk of developing arthritis in the affected joints in the future, so even if a patient is able to return to sport, they may have long-term sequelae in the future.
Question 28 of 100
Figures 1 and 2 are the radiographs of a 32-year-old woman who has right foot pain after falling down a few steps. For the best long-term outcome, initial treatment should include
A. splinting with non-weight-bearing as the definitive treatment.
B. closed reduction and casting.
C. percutaneous pinning.
D. primary open reduction and internal fixation (ORIF).
R: D
The radiographs show a displaced Lisfranc injury. The outcome of treatment is dependent on achieving an anatomic reduction and stabilization, which is only possible with primary ORIF. Some studies indicate primary fusion may provide superior short-term results compared with ORIF. Closed treatment (reduction with casting or splinting) will not achieve or maintain the reduction, whereas delayed treatment by secondary fusion after arthritis occurs yields inferior outcomes to primary ORIF.
Question 29 of 100
Figure 1 is the radiograph of a 25-year-old soccer player who twisted her right ankle 1 week ago. She has pain and swelling over the anterolateral ankle and there is ecchymosis over the lateral ankle. She has these muscle group findings: anterior tibial tendon-right 5/5, left 5/5; posterior tibial tendon-right 5/5, left 5/5; peroneals-right 4/5, left 5/5; Achilles-right 5/5, left 5/5. What is the best next diagnostic or treatment step?
A. Stress radiographs
B. Surgical resection of the fragment with lateral ligament reconstruction
C. Physical therapy for peroneal strengthening and proprioceptive training D. Ankle arthroscopy for debridement
R: C
Thousands of ankle sprains occur in the United States every day. Most affected patients do not have serious sequelae associated with their injury. In this case, a young athlete sprained her ankle. Her only area of tenderness is isolated to the anterior talofibular ligament. She also has associated weakness. The radiograph shows an os subfibulare; this is an entity that she likely was born with. There is no indication of bony pain, and it is too soon to test for instability; consequently, no further imaging is required. Considering the nature of the sprain and her weakness, physical therapy with proprioceptive training and peroneal strengthening would be most beneficial.
Question 30 of 100
Talonavicular arthrodesis is used to treat symptomatic isolated talonavicular arthritis. Fusion of the talonavicular joint decreases motion of the hindfoot joints by approximately what percentage of the prearthrodesis range of motion?
A. 10%
B. 25%
C. 50%
D. 90%
R: D
Arthrodesis of the talonavicular joint decreases overall motion of the hindfoot joints to <8% of the presurgical range of motion.
Question 31 of 100
A 35-year-old man sustained a Lisfranc dislocation 2 years ago. He was treated with standard open reduction and fixation. At 4 months, the screws were removed. He now has increasing pain and discomfort. A current radiograph is shown in Figure 1. What is the best treatment option?
A. Reduction and fusion of the medial three tarsometatarsal (TMT) joints
B. Reduction and fusion of all five tarsometatarsal joints
C. Revision open reduction and internal fixation with bridge plates to avoid further damage to the joints D. Soft-tissue interpositional grafts for the tarsometatarsal joints
R: A
The most reliable treatment will be a reduction and fusion of the medial three TMT joints. There is adequate proof in the literature that fusion of all five TMT joints should be avoided because the fusion rate as well as functional outcome is inferior with fusion of all five joints compared with fusion of the medial three TMT joints and preservation of mobility in the 4-5 TMT joints. There is too much deformity and arthritis to warrant a revision open reduction and internal fixation. An interpositional graft is not proven to help in this situation because it neither corrects the deformity nor aids in stability.
Question 32 of 100
A 39-year-old woman who was previously involved in a motor vehicle collision status post closed intraarticular fracture to the distal tibia. Two years after open reduction and internal fixation, she has persistent ankle pain during activities of daily living, despite the use of a custom leather gauntlet with radiographs shown in Figures 1 and 2. Examination reveals tenderness to palpation along the ankle joint with no pain in the hindfoot. Range of motion is noted at -5° dorsiflexion, 10° plantar flexion, 5° inversion, and 5° eversion. The incision is well-healed and anterolaterally based. Decreased sensation is noted to the superficial peroneal nerve. The surgical intervention with the most reliable pain relief and lowest risk of reoperation for this patient is
A. distraction arthroplasty with an articulated external fixator.
B. ankle arthroplasty and Achilles tendon lengthening.
C. hardware removal with anterior tibiotalar cheilectomy.
D. ankle arthrodesis.
R: D
This is a younger patient with posttraumatic ankle arthritis secondary to a pilon fracture. She has limited range of motion with no hindfoot symptoms. Nonsurgical management would include the use of an Arizona
brace or ankle-foot orthosis, in addition to nonsteroidal anti-inflammatory drugs and corticosteroid injections. If those modalities do not offer relief, surgical intervention is appropriate.
Although distraction arthroplasty can be considered and has demonstrated some limited success in patients who may be considered too young or active for either a fusion or arthroplasty, this is not a reliable option and is not the best answer given the other options. Ankle arthroplasty with Achilles tendon lengthening is a reasonable option, and recent data show that in the scenario of high-energy trauma with resultant arthritis and in limited preoperative range of motion, such patients may benefit from an ankle arthroplasty. However, given her young age, (39 years-old), she will be at a high risk for further revision of the implant. If she has concomitant hindfoot arthritis or hindfoot arthrodesis, this would be a more reasonable option to consider. Hardware removal with tibiotalar joint cheilectomy will not solve the patient's complaints and is only appropriate for patients with complaints of impingement, not chronic joint pain secondary to arthritis. In this patient, her younger age, limited range of motion, and lack of hindfoot arthritis indicate that the best surgery with the least risk of reoperation is an ankle arthrodesis. Although patients will develop adjacent joint disease in the future, many do not require further surgery, in contrast to a 39-year-old patient with an ankle arthroplasty, given her expected life expectancy in whom the need for a revision ankle arthroplasty is nearly guaranteed.
Question 33 of 100
A 45-year-old man has a grade 4 hallux rigidus secondary to a turf toe sustained as a football player in high school. He is an avid golfer and plays tennis on occasion. His activities are severely limited because of pain in his great toe and nonsurgical management has failed to provide relief. His goal is to be pain free, continue with his activities, and require no further orthopaedic care in the future. What is the best treatment option for this patient?
A. Keller's excision arthroplasty
B. Cheilectomy and débridement
C. Great toe metatarsophalangeal (MTP) fusion
D. Fascia lata interposition graft
R: C
There has been some research about interpositional grafting, whether autologous or allograft, but there is no convincing evidence at this point that there is a better functional outcome than with a great toe MTP fusion. A Keller procedure is not indicated for a young, active person. A joint replacement of the great toe similarly has not proven to be a reliable option for younger, active people. Cheilectomy will not provide reliable pain relief in grade 4 arthrosis of the first MTP joint.
Question 34 of 100
Which radiographic abnormality most accurately serves as a predictor of ankle syndesmosis disruption?
A. Medial clear space equal to the superior clear space on the anteroposterior (AP) view
B. Tibiofibular overlap exceeding 6 mm on the AP view
C. Tibiofibular clear space exceeding 6 mm on AP view
D. Talocrural angle symmetric to the opposite side
R: C
Normal syndesmotic relationships include a tibiofibular clear space smaller than 6 mm on both AP and mortise views. In a 1989 cadaveric study by Harper and Keller, a tibiofibular clear space exceeding 6 mm on both the AP and mortise views was the most reliable predictor of early syndesmotic widening. Tibiofibular overlap is measured 1 cm proximal to the plafond. Normal values exceed 6 mm or 42% of the width of the fibula on the AP view, or 1 mm on the mortise view. Proximal fibula fracture can occur in isolation without syndesmotic injury, frequently after direct trauma. The medial clear space is the distance between the lateral border of the medial malleolus and the medial border of the talus and is measured at the level of the talar dome. In the mortise view with the ankle in neutral dorsiflexion, the medial clear space should be equal to or smaller than the superior clear space between the talar dome and the tibial plafond. A normal medial clear space may be present with syndesmotic injury and consequently lacks sensitivity and specificity.
Question 35 of 100
Figure 1 shows the recurrent ulcer of a diabetic patient with a toe brachial index of >0.5 and a hemoglobin A1C of 6.8. What can be done to reduce the recurrence of this problem?
A. Endovascular procedure to improve blood flow
B. Referral to endocrinologist
C. Achilles tendon lengthening
D. Application of a walking boot
R: C
Reduction in sensation prevents diabetic patients with neuropathy from detecting injury to the foot. Changes in the Achilles tendon from glycosylation results in a contracted and stiff tendon, which produces increased pressure over the plantar forefoot. An ankle-brachial index (ABI) <0.45 would be indicative of poor healing potential and the need for vascular intervention. If the ABI is falsely elevated, toe pressures can be used instead. A glycosylated hemoglobin level of ≥8 is a concern for increased complication for surgical procedures. Evidence-based guidelines give the following grade A for preventing recurrence: custom shoes/ orthotics and Achilles tendon lengthening.
Question 36 of 100
Figure 1 is the radiograph of a patient with type 2 diabetes, a body mass index of 42, and an Hgb A1c of 8. What is the most appropriate management for this injury?
A. Casting the ankle in its current position
B. Closed reduction and definitive casting
C. Closed reduction and application of external fixation
D. Open reduction and internal fixation (ORIF)
R: D
Several recent studies have shown that while there is an increased risk of complications following ORIF of displaced ankle fractures in diabetic patients compared with nondiabetic patients, the overall risks of treatment are less than that associated with nonsurgical treatment in diabetics. There is also the possibility that ORIF of unstable ankle fractures may forestall the development of Charcot changes in the ankle, although this is not definitively known. Extra rigid fixation may be required because of the patient's size and poorly controlled diabetes. Nonsurgical management is associated with poorer functional outcomes (due to arthritis secondary to poor reduction of the fracture) and a higher rate of skin breakdown, due to the need for higher skin pressures from the use of highly molded casting used to maintain a closed reduction.
Question 37 of 100
A 55-year-old woman has pain localized to her left plantar forefoot. The pain is worse when walking barefoot or wearing high heels. Physical examination demonstrates tenderness in the third greater than the second intermetatarsal space, with palpable click and paresthesia in the third interspace only with lateral compression of the metatarsals. Her radiographs are unremarkable. What is the most appropriate surgical management?
A. Release of transverse metatarsal ligament in the third webspace
B. Release of transverse metatarsal ligament in second and third webspace
C. Excision of neuroma in the third webspace
D. Excision of neuroma in the third webspace and release of transverse metatarsal ligament in the second webspace
R: D
Intermetatarsal neuritis, commonly referred to as Morton's neuroma, is most commonly related to recurrent trauma to the plantar forefoot resulting in injury to the third intermetatarsal nerve (2/3) or second intermetatarsal nerve (1/3). This is frequently related to poorly fitting shoes or recurrent trauma from running. Diagnosis is primarily clinical and is characterized by tenderness in the intermetatarsal space and possibly a palpable click seen with lateral compression of the metatarsals with thumb pressure to the plantar intermetatarsal space (Mulder’s sign). Diagnostic lidocaine injection can also be helpful to determine which interspace is involved. Nonsurgical treatment involves use of wide toe box shoes, avoidance of impact to forefoot, metatarsal pad, and corticosteroid injection. Surgical management can be considered if nonsurgical management fails and should involve excision of the offending intermetatarsal nerve. In this case, the patient has pain in two adjacent web spaces with the third more symptomatic than the second. Preferred treatment would involve excision of the third intermetatarsal nerve and decompression of the second through release of the transverse metatarsal ligament between second and third metatarsal. Release of the ligament alone is less likely to improve the pain.
Question 38 of 100
Figure 1 is the clinical photograph of a 42-year-old woman with type 2 insulin-dependent diabetes who has a 1-week history of redness and swelling along the forefoot. The patient has been admitted to the hospital and is stable and afebrile on intravenous (IV) antibiotics. Her erythrocyte sedimentation rate is 70 mm/h (reference range [RR], 0 to 20 mm/h), C-reactive protein level is 5.4 mg/L (RR, 0.08 to 3.1 mg/L), white blood cell count is 11.5 (RR, 4,500 to 11,000 /µL), and her albumin level is 1.5. The examination demonstrates an inability to sense a 5.07 Semmes-Weinstein monofilament and 1+ dorsalis pedis and posterior tibialis pulses. What is the best next step?
A. Observation on continued IV antibiotic treatment
B. Emergent transmetatarsal amputation
C. Urgent surgical debridement
D. Noninvasive arterial flow studies
R: C
This is a neuropathic diabetic patient who presents with a clear infection and abscess of the great toe and surrounding tissue. The neuropathy is the primary cause of why diabetic patients do not complain of pain and present with infections at late stage, with significant swelling. The significance of the infection and soft-tissue damage may be worse than the patient’s subjective symptoms suggest. The patient is currently on IV antibiotics, which is a common scenario in the inpatient hospital setting. Antibiotic suppression of the infection and diabetic immunocompromise commonly result in lack of a febrile response. The laboratory values, specifically the albumin level of 1.5, suggest the patient is nutritionally compromised and does not have the ability to heal the wound. Hypoalbuminemia (albumin, <3.5 g/dL) is an independent risk factor for infectious and wound healing complications. Continued IV antibiotics is not appropriate because this course will not definitively treat the abscess. Obtaining arterial blood flow studies should not delay surgical debridement. The patient has 1+ palpable pulses. Emergent amputation is not appropriate at this time because of the stable clinical scenario. In the unstable patient, emergent guillotine amputation would be appropriate to eliminate the infectious source.
Question 39 of 100
Figure 1 is the radiograph of a 46-year-old woman who has had several years of foot pain. She has demonstrated only minimal relief with boot immobilization, orthotic use, and physical therapy. She elects for surgical correction of her deformity. In addition to osteotomies and a flexor digitorum longus tendon transfer, which other soft-tissue procedure may be necessary based on an intraoperative examination?
A. Gastrocnemius recession
B. Peroneous longus to brevis tendon transfer
C. Posterior tibialis tendon transfer to dorsum of foot
D. Flexor hallucis longus tendon transfer to calcaneus
R: A
The patient is diagnosed with adult acquired flatfoot deformity. Surgery often involves a medial displacing calcaneal osteotomy, flexor digitorum longus tendon transfer, medial cuneiform osteotomy (Cotton osteotomy), and sometimes a lateral column lengthening. As the hindfoot moves into a hindfoot valgus position, the gastrocnemius and/or the Achilles tendon develop a contracture. To obtain the appropriate deformity correction, the contracture needs to be released. A peroneus longus to brevis tendon transfer is often used in a flexible cavovarus foot. A posterior tibialis tendon transfer to the dorsum of the foot is used for drop foot. A flexor hallucis longus tendon transfer is often used for chronic Achilles tendon.
Question 40 of 100
A 50-year-old man is having difficulty walking. He has a history of an injury 6 months ago. Examination reveals that he is unable to toe off and has increased ankle dorsiflexion compared with the uninjured side. He has a positive Thompson test and a palpable defect in the Achilles tendon. An MRI scan shows a 4-cm defect in the Achilles tendon. What type of surgical reconstruction should be planned?
A. V-Y advancement with or without flexor hallucis longus augmentation
B. Flexor hallucis longus tendon transfer
C. Flexor digitorum longus tendon transfer
D. Peroneus brevis tendon transfer
R: A
The patient has a chronic Achilles tendon rupture with a 4-cm defect. A V-Y advancement avoids sacrifice of a normal muscle tendon unit and will fill a defect of 2 cm to 5 cm. The flexor hallucis longus may be used to augment a V-Y advancement, particularly if there is significant scarring or atrophy of the gastrocnemius soleus complex. A flexor hallucis longus tendon transfer is indicated for defects of greater than 5 cm. Peroneus brevis tendon transfer is able to augment the Achilles tendon but will compromise eversion strength.
Question 41 of 100
A 70-year-old patient has a history of midfoot pain. The onset occurred over a several-year period and has progressively worsened. The pain is described as aching and is provoked by activities and is improved with rest. Some swelling is also noted. The examination shows normal appearance of the foot and ankle. Resisted dorsiflexion of the ankle is painless, with 5/5 strength noted. Sensation and pulses are intact. Tenderness is noted about the medial midfoot. His radiographs are shown in Figures 1 through 3. What is the best next step in management?
A. Debridement of the anterior tibialis tendon
B. Arthrodesis of the medial column
C. Morton's extension orthotic
D. Rocker bottom shoes
R: D
The patient presents with symptoms, and physical and radiographic examination consistent with midfoot arthritis. Nonsurgical management often consists of a trial of activity modifications, use of nonsteroidal antiinflammatory drugs, and shoe modifications such as rocker bottom soles. Morton’s extension orthotics immobilize the first ray and are indicated for hallux rigidus or sesamoid injuries. Arthrodesis of the medial column does not address the arthritic changes noted in the lesser metatarsal tarsal joints. The absence of pain with resisted dorsiflexion of the ankle makes tendonitis of the anterior tibialis tendon unlikely.
Question 42 of 100
A 45-year-old woman with grade II adult-acquired flatfoot deformity has pain on the lateral side of her foot just distal to the tip of the fibula. Which component of a comprehensive flatfoot reconstruction most likely will address the deformity responsible for this pain?
A. Spring ligament reconstruction
B. Lateral column lengthening
C. Medial-displacing calcaneal osteotomy
D. Medial cuneiform opening-wedge osteotomy
R: C
Patients develop lateral ankle pain with progression of adult-acquired flatfoot deformity. This is associated with increased hindfoot valgus deformity. Calcaneal fibular impingement has been considered the primary cause of this pain. Studies demonstrate that arthrosis of the posterior facet of the subtalar joint strongly correlates with lateral pain in adult-acquired flatfoot deformity. Both conditions are related to hindfoot valgus deformity. Although lateral column lengthening is a powerful tool for correction of flatfoot deformity, its effect on hindfoot deformity is less defined. Lateral column lengthening provides better correction of the longitudinal arch of the midfoot and realignment of the medial column than other osteotomies. A medializing calcaneal osteotomy has a significant linear effect on hindfoot valgus alignment. Spring ligament reconstruction and medial cuneiform opening-wedge osteotomies have less effect on hindfoot alignment than the medial calcaneal slide.
Question 43 of 100
A 22-year-old woman has physical examination findings consistent with an isolated Morton’s neuroma in the third webspace. Treatment with inserts and shoe modification fails to resolve the problem. She subsequently undergoes a series of three steroid injections. She has temporary relief of pain after each injection; however, 3 months after the injection, she has continued pain. What is the best next step?
A. MRI scan to evaluate for other causes of pain
B. Excision of the Morton’s neuroma
C. Repeat series of steroid injections
D. Oblique osteotomy of the third metatarsal
R: B Corticosteroid injections and placebo injections both have about a 50% effectiveness in treating Morton’s neuroma. To maximize the placebo effect, a series of three closely spaced injections has been recommended. The potential for placebo effect after initial treatment is diminished. Failure of the injections is an indication to proceed with operative treatment. Studies have not documented that MRI is effective at identifying associated conditions, and it is not superior to physical examination on identifying a Morton’s neuroma.
Question 44 of 100
Figures 1 and 2 are the AP and lateral radiographs of a 65-year-old woman who has a dislocated second toe and a prominent bunion. Besides repairing the bunion, what procedures are recommended to address the fixed second hammertoe and the resulting metatarsalgia?
A. Proximal interphalangeal joint (PIP) resection arthroplasty
B. PIP resection arthroplasty, extensor tendon lengthening, and Weil osteotomy
C. Flexor to extensor tendon transfer and metatarsal head excision D. PIP fusion and DuVries arthroplasty
R: B
The patient has a subluxated second metatarsophalangeal (MTP) joint, but no evidence of second metatarsal head destruction. The patient also has a fixed claw toe. The claw toe is repaired with a PIP resection arthroplasty or a PIP fusion. The likelihood of completely correcting the MTP joint dislocation with just these two procedures, however, is small and the addition of the Weil osteotomy is more likely to allow decompression of the joint and complete relocation of the MTP joint. The flexor to extensor transfer is indicated for a flexible hammertoe. A metatarsal head excision is a salvage option of the toe that still cannot be reduced after the Weil osteotomy, but this may lead to transfer lesions. The DuVries arthroplasty will not help reduce the MTP joint. Combined metatarsal head excision and proximal phalanx resection would be extreme and is not recommended.
Question 45 of 100
Figures 1 and 2 are the radiographs of a 57-year-old woman who has pain localized to the right hallux metatarsophalangeal (MTP) joint dorsally. Examination demonstrates 40° dorsiflexion and 10° plantarflexion of the hallux MTP joint. There is pain at terminal dorsiflexion but no pain at midrange.
She has not responded to shoe modifications and intra-articular corticosteroid injection. What is the most appropriate surgical treatment for her metatarsophalangeal condition at this time?
A. Cheilectomy
B. Interpositional arthroplasty
C. Total joint arthroplasty
D. Arthrodesis
R: A
The diagnosis in this patient is grade 1 hallux rigidus, which is characterized radiographically by minimal spurring and preserved joint space and clinically by loss of dorsiflexion range of motion with pain at terminal dorsiflexion but no midrange pain. Radiographically, the patient demonstrates mild joint space narrowing with dorsal and marginal osteophyte formation. Treatment of choice is cheilectomy. In the absence of midrange pain, neither arthrodesis nor total joint arthroplasty is advised. Total joint arthroplasty also has not shown satisfactory long-term outcomes. Interpositional arthroplasty can be considered as an alternative to fusion in a patient with end-stage hallux rigidus with midrange pain; however, long-term results are poor.
Question 46 of 100
Figures 1 and 2 are the current AP and oblique radiographs of a 44-year-old man who underwent nonsurgical management of a left ankle fracture 6 months ago. What is the most appropriate course of management?
A. Arizona brace
B. Ankle arthroscopy with drilling of the talar osteochondral lesion
C. Medial ankle arthrotomy and debridement with correction of the fibular malunion D. Ankle and subtalar arthrodesis
R: C
The radiographs reveal a fractured malunited, shortened fibula with deltoid instability. Corrective osteotomy with fibular lengthening has shown positive results. Nonsurgical management in an active, healthy patient will lead to rapid deterioration of the ankle joint. Without evidence of arthritis, a joint-sacrificing procedure should not be used.
Question 47 of 100
Eight months following an inversion injury to her ankle, a 32-year-old woman still has persistent pain with activity along the posterolateral ankle. The use of a lace-up ankle brace and physical therapy has not helped. She notes a snapping sensation along the lateral ankle that is associated with pain during activity. On examination, apprehension is noted with resisted eversion that is relieved with pressure along the posterolateral border of the fibula. Ankle stress testing reveals stability to both anterior drawer and inversion stress testing. Radiographs are negative for pathology. What is the most appropriate intervention to relieve her symptoms?
A. Short leg cast for 6 weeks followed by functional rehabilitation
B. Flexor hallucis tendon transfer to the base of the fifth metatarsal
C. Fibular groove deepening with imbrication of the peroneal retinaculum D. Brostrom reconstruction with Gould modification
R: C
The patient has clear evidence of peroneal tendon subluxation. This can be either intrasheath or frank subluxation. In both pathologies, the patients will note a snapping sensation along the peroneal tendons. In cases of frank subluxation, the patient will have apprehension to resisted eversion, as this will cause the subluxation. In some chronic cases, the patient can subluxate the tendons with minimal pain and can reproduce the subluxation in clinic. Nonoperative management for chronic peroneal subluxation has minimal benefit and provides no long-term resolution for their pain. Additionally, repetitive subluxation is thought to increase the risk of degenerative pathology to the tendons, resulting in longitudinal tears, more commonly of the brevis. Therefore, in this case, the patient is best treated with a groove deepening and imbrication of the peroneal retinaculum to eliminate the subluxation. There is no evidence of instability, therefore, a Brostrom reconstruction has no role in this case, but in many patients, associated instability is noted and should be treated simultaneously. A flexor hallucis longus transfer to the base of the fifth metatarsal is utilized for revision peroneal tendon pathology, where no viable muscle is present, and this procedure is performed as a salvage to provide some dynamic stability to the ankle.
Question 48 of 100
Figure 1 is the AP radiograph of a 59-year-old woman with chronic plantar medial pain referable to the first metatarsosesamoidal joint of her right foot for which she subsequently underwent corrective surgery. Figure 2 is the postoperative AP radiograph. What progressive forefoot deformity may arise secondary to the selected surgical procedure?
A. Metatarsus primus elevatus
B. Metatarsus atavicus
C. Hallux varus
D. Hallux valgus
R: D
Surgical medial sesamoidectomy can lead to the development of hallux valgus deformity. Metatarsus primus elevatus refers to fixed dorsal inclination of the first metatarsal in relation to the lesser metatarsals, whereas metatarsus atavicus describes relative shortness of the first metatarsal compared with the adjacent second metatarsal length. Excision of the lateral sesamoid may predispose to hallux varus deformity.
Question 49 of 100
A 49-year-old woman underwent a successful right ankle fusion. She now reports an altered gait. In an attempt to improve her gait, what is the most appropriate device?
A. Arizona brace
B. Rocker-bottom sole
C. Double upright drop-lock brace
D. Non-articulated ankle-foot orthosis
R: B
The plantar flexion-dorsiflexion motion of the ankle can be partially mimicked with a rocker-bottom shoe adaptation. An ankle-foot orthosis, Arizona brace, and double upright drop-lock brace would immobilize the ankle, which is already achieved with the ankle fusion.
Question 50 of 100
Figures 1 and 2 show the radiographs of a 44-year-old woman who has chronic lateral foot and ankle pain. What structure is primarily responsible for the plantarflexion deformity of the first metatarsal?
A. Flexor hallucis longus
B. Posterior tibial tendon
C. Plantar fascia
D. Peroneus longus tendon
R: D
Cavovarus foot deformities tend to present with complaints of chronic ankle sprains, peroneal tendon pathology, and lateral column foot pain. Often this deformity is due to a muscle imbalance in which the peroneus longus overpowers the anterior tibial tendon, causing plantarflexion of the first ray and the posterior tibial tendon overpowers the peroneus brevis, contributing to the adduction at the hindfoot and midfoot. The posterior tibial tendon does not contribute to the first metatarsal deformity. Additionally, the flexor hallucis longus is typically overpowered by the extensors, which contributes to a clawtoe deformity but not plantarflexion of the first metatarsal. The plantar fascia is often released during cavovarus correction; however, this contributes more to the deformity at the midfoot. After correcting the hindfoot varus, forefoot pathology should be addressed. If there is continued plantarflexion of the first metatarsal head relative to the lesser metatarsal heads, the peroneus longus is released and tenodesed to the peroneus brevis. This will also help diminish recurrence of the deformity, particularly in cases of muscular imbalance such as Charcot-MarieTooth.
Question 51 of 100
A 40-year-old runner has a flexion deformity of the distal interphalangeal (DIP) joint of his second toe. He has pain at the tip of the toe, particularly during longer runs. He has tried multiple pads and shoes with a wider toe box without relief. On examination, the deformity is flexible and passively correctible. The best surgical option is
A. DIP arthroplasty.
B. DIP fusion.
C. flexor digitorum longus (FDL) tenotomy.
D. FDL transfer to extensor hood.
R: C
A mallet toe is a flexion deformity at the level of the proximal interphalangeal joint of the second toe. These can be flexible or rigid. Flexible deformities can be treated with flexor tenotomies. Rigid deformities typically require a DIP joint fusion. The FDL transfer to the extensor hood is typically done to correct a flexible proximal interphalangeal joint deformity (hammer toe).
Question 52 of 100
Figure 1 represents the radiographs of a 12-year-old girl with medial foot pain. She has a physiologic hindfoot alignment without evidence of pes planus. Her pain has persisted, despite the use of orthotics, nonsteroidal anti-inflammatory drugs, and 4 weeks in a cast. What is the best next step?
A. Open reduction and internal fixation (ORIF) of the navicular fracture
B. Excision of accessory navicular with reattachment of the posterior tibialis tendon
C. Steroid injection into the posterior tibial tendon
D. Medial displacing calcaneal osteotomy and flexor digitorum tendon transfer
R: B
This question involves an adolescent girl with an accessory navicular. She is symptomatic and has failed to improve with the appropriate nonoperative treatment. In the absence of pes planus, the best option includes excision of the accessory navicular and reattachment of the posterior tibialis tendon into the defect. If the patient has a flatfoot deformity, this deformity would need to be addressed concomitantly. This patient does not have a fracture, so ORIF is not indicated.
Question 53 of 100
A 72-year-old woman with a moderately reducible hallux varus has pain in the first metatarsophalangeal (MTP) joint that is activity related and reports that she cannot find any comfortable shoes. She wants to know what treatment plan offers her the most predictable outcome in terms of pain relief, activity, and the ability to get into shoes?
A. First MTP fusion
B. MTP joint replacement
C. Keller resection arthroplasty
D. Tendon transfer and capsular release
R: A
A great toe fusion is the most appropriate treatment. It is an excellent procedure for pain relief and it gives a predictable result for return to activity and lack of recurrence. A soft-tissue correction is not indicated due to patient age and reducibility. The Keller resection arthroplasty and the MTP joint replacement allow motion, but they offer unpredictable results for pain relief, activity, and recurrence.
Question 54 of 100
Figure 1 is the MRI scan of a 64-year-old man who has had left ankle pain for 3 weeks following increased running activity and now has residual antalgia. Physical examination reveals medial-sided tenderness and swelling of his left ankle. What is the most appropriate treatment at this time?
A. Custom shoe inserts
B. Physical therapy services
C. Open reduction and internal fixation
D. Cam boot immobilization
R: D
The MRI scan reveals incomplete vertically oriented stress fracture of the medial malleolus with associated marrow edema. Although complete stress fractures of the medial malleolus are often surgically treated, incomplete fractures are most appropriately treated with strict immobilization (cast or Cam boot) and protected weight-bearing status. Physical therapy, custom shoe inserts, and change in training regimen are more applicable once the stress fracture has healed.
Question 55 of 100
Figure 1 is the clinical photograph of a 54-year-old man who underwent a total ankle replacement (TAR). Three weeks after surgery he has increasing pain and a deep wound as seen in the photograph. What is the best next step?
A. Remove the total ankle and place an antibiotic spacer
B. Debride and exchange polyethylene
C. Perform a below-the-knee amputation
D. Convert to a fusion with an intercalary allograft
R: B
The patient is 3 weeks out from TAR. The wound is erythematous, and the tendon is visible. At 3 weeks this is an acute wound breakdown. The preferred treatment is a return to the operating room, an exchange of the polyethylene because the wound appears deep enough to go down to the joint, and a flap for coverage. Removal of the total ankle and placement of an antibiotic spacer should be considered in the settings of subacute (6 weeks postop) or chronic infection following TAR. A below-the-knee amputation may be considered with a failed salvage or a chronically infected TAR. Conversion to a fusion may be considered in situations in which the wound bed is not infected. In this case, there is concern for ongoing active infection, and an intercalary allograft is not appropriate.
Question 56 of 100
A 25-year-old woman is seen for worsening forefoot pain. The pain began about 4 weeks prior after the patient doubled the distance of her daily jogging routine. On examination, there is mild swelling noted throughout the forefoot. No tenderness is noted along the metatarsals. Pain is elicited with passive plantarflexion of the second toe. The foot is warm and well perfused. The finding of pain with passive flexion is consistent with what diagnosis?
A. Metatarsal stress fracture
B. Rheumatoid arthritis
C. Freiberg’s infraction
D. Gastrocnemius contracture
R: C
Passive flexion of the metatarsophalangeal (MTP) joint indicates intra-articular pathology (i.e., synovitis). A metatarsal stress fracture should be considered in this patient, but this diagnosis would not routinely cause MTP synovitis. A gastrocnemius contracture may predispose the patient to forefoot pathology but would not directly explain this physical examination finding. Rheumatoid arthritis can be associated with MTP pathology, but in this clinical setting is not probable. The age, gender, and activity of this patient are all risk factors for developing Freiberg’s infraction.
Question 57 of 100
The MRI scan of the ankle shown in Figure 1 reveals a tear of what structure?
A. Superficial peroneal retinaculum
B. Posterior tibial tendon
C. Peroneus brevis tendon
D. Anterior talofibular ligament (ATFL)
R: C
The MRI scan shows a tear of a tendon behind the fibula. The peroneus brevis is deep to the longus (closer to the fibula). There should only be two structures behind the fibula, but the peroneus brevis tendon is shown as two distinct structures which is clearly a longitudinal tear. The posterior tibial tendon would be posteromedial to the tibia. The superficial peroneal retinaculum is behind the fibula, but would show up as a tear if the peroneal tendons were dislocated, or lateral to the fibula. The ATFL runs from the anterior fibula to the talus.
Question 58 of 100
Figure 1 is the radiograph of a 65-year-old man who has a painful deformity 15 months after a bunionectomy. On examination, his metatarsophalangeal (MTP) joint has 40° of painful motion. What procedure will most reliably correct his problem?
A. Hallux MTP arthrodesis
B. Suture button reconstruction of the medial collateral ligament
C. Extensor hallucis longus split tendon transfer
D. Takedown and revision of the lapidus fusion
R: A
Following hallux valgus surgery, one potential complication is the development of hallux varus. Mild deformities can sometimes be managed with wider toe boxes and splinting/taping. More significant deformities require surgical correction. In the absence of arthritic changes, soft-tissue realignment procedures can be considered. These include extensor hallucis longus tendon transfers and suture button techniques. If the joint has developed stiffness and arthritis as in this patient, then the preferred treatment is a MTP arthrodesis.
Question 59 of 100
Figure 1 is the radiograph of a laborer who has hindfoot and ankle pain. He is a type 1 diabetic, and has a BMI of 25 and a Hgb A1c of 6. What is the most appropriate management at this time?
A. Total contact casting
B. Open reduction and internal fixation
C. Bed rest
D. Standard walking boot
R: A
The radiograph shows Charcot changes in the subtalar joint. In the absence of gross deformity, the initial treatment is nonsurgical, consisting of total contact casting, with frequent cast changes and progression to weight bearing when swelling subsides and early consolidation is seen radiographically. A walking boot will not provide sufficient immobilization, whereas bed rest carries the risk of significant deconditioning in an otherwise active patient with well-controlled diabetes. Surgery as the initial treatment is not indicated in the absence of gross deformity or ulceration.
Question 60 of 100
A 12-year-old boy has had medial midfoot pain with activity over the last few months. Radiographs show a large type II accessory navicular. He has a flexible flatfoot deformity. What would be expected to be seen on an MRI scan?
A. Plantar fascia tear
B. Bone marrow edema in the accessory navicular
C. Thickening of the spring ligament
D. Sprain of the Lisfranc ligament
R: B
Patients with an accessory navicular that becomes symptomatic are often participating in sports or sustain a low level of trauma such as a sprain. MRI scan will show thickening of the posterior tibial tendon insertion, marrow edema in the accessory navicular, marrow edema in the navicular tuberosity, and contrast enhancement at the posterior tibial tendon insertion.
Question 61 of 100
Figures 1 and 2 are the radiographs and MRI scan of a 17-year-old cross country runner who reports pain in his forefoot around the third and fourth metatarsals. The pain is mostly on top of the foot and appears to be activity related. There is minimal swelling on examination and diffuse tenderness over the third and fourth metatarsal shafts. What is the most appropriate next step in managing his pathology?
A. Three-phase bone scan
B. Non-weight-bearing short-leg cast
C. Fracture boot with weight bearing as tolerated
D. Limit his miles and repeat radiographs in 2 weeks
R: C
The most appropriate management is a fracture boot with weight bearing as tolerated. The radiographs are normal, but the history strongly suggests a metatarsal stress fracture. The MRI scan clearly shows edema of the third metatarsal which is consistent with a stress fracture. Patients can be treated successfully with weightbearing immobilization and activity modification. Some physicians treat metatarsal base fractures with limited weight bearing. Limiting his miles and repeat radiograph in 2 weeks would be an acceptable option if the MRI scan showing the fracture had not been obtained. A bone scan is a good test to diagnose the fracture, but an MRI scan has already been obtained.
Question 62 of 100
Figures 1 through 3 reveal the weight-bearing foot and ankle radiographs, respectively, of a 65-yearold woman with history of obesity and chronic right foot pain with progressive flatfoot deformity present for several years. Examination demonstrates a rigid planovalgus foot deformity with hindfoot valgus, forefoot abduction, and collapse of medial longitudinal arch. She has inversion weakness and inability to perform a single limb heel raise. Passive dorsiflexion of the ankle in subtalar neutral position is approximately 0°. She has failed nonsurgical management with orthotics and bracing. Surgical management of this condition would include
A. Medial displacement calcaneal osteotomy and flexor digitorum longus transfer
B. Lateral column lengthening calcaneal osteotomy, flexor digitorum longus transfer, and gastrocnemius recession
C. Triple arthrodesis with gastrocnemius recession
D. Tibiotalocalcaneal arthrodesis
R: C
This case demonstrates a patient with stage III adult acquired flatfoot deformity. In stage III, the deformity is rigid and not passively correctible. It is important to distinguish between stage III and stage IV, in which there is deltoid ligament insufficiency and resultant valgus talar tilt. In this case, the patient is not a candidate for joint-sparing procedures due to the severity and rigidity of her deformity and as such, triple arthrodesis would be the treatment of choice, along with gastrocnemius recession to address the equinus. Given that the tibiotalar joint appears unaffected on weight-bearing ankle radiographs, a tibiotalocalcaneal fusion is not indicated at this time.
Question 63 of 100
Figures 1 and 2 are the weight-bearing radiographs of a 17-year-old girl who has great toe pain with push-off and stiffness 1 year after undergoing a proximal crescentic osteotomy for hallux valgus. Motion at the first metatarsophalangeal joint includes approximately 20° of dorsiflexion. What is the most appropriate treatment?
A. Proximal phalanx osteotomy
B. Plantar flexion metatarsal osteotomy
C. Distal biplanar metatarsal osteotomy
D. Capsular release and aggressive physical therapy
R: B
The patient has progressed to a dorsiflexion malunion of the first metatarsal. The absence of implants suggests that smooth pin fixation was likely used, likely contributing to the malunion. The patient should be managed with a plantar flexion metatarsal osteotomy. The joint stiffness is likely a result of the malunion acting as a mechanical block to dorsiflexion, thus a capsular release would not be sufficient. The distal biplanar metatarsal osteotomy is used for hallux valgus deformities associated with an increased distal metatarsal articular angle. The proximal phalanx osteotomy is used for supplemental correction of associated hallux valgus interphalangeus.
Question 64 of 100
A 25-year-old female collegiate athlete sustains an inversion injury to the right ankle during soccer. She denies prior injury to the affected leg. Seven days after the injury, she has ecchymosis and swelling along the lateral ankle without proximal tenderness. She is able to bear weight without an assistive device. A midcalf squeeze test and external rotation stress test are negative. Tenderness to palpation is noted at the inferior tip of the fibula and immediately distal. Anterior drawer testing reveals 1+ drawer compared with the contralateral lower extremity. Radiographs of the ankle are negative for fracture. What is the best next step?
A. Lace-up ankle brace and physical therapy for peroneal strengthening and proprioception
B. MRI scan to determine the extent of soft-tissue injury to guide initial management
C. Primary repair of the lateral collateral ligaments
D. Non-weightbearing with use of a controlled ankle movement (CAM) boot
R: A
This is a collegiate-level athlete with a low ankle sprain. She has no prior history of recurrent instability and lack of clinical evidence of a syndesmotic injury. Although MRI scan can be performed to identify the extent of the disruption of the lateral collateral ligaments and can aid in grading, the initial treatment does not necessitate MRI; and therefore, advanced imaging is not the most appropriate answer. The patient can bear weight, and there is no clinical evidence of a peroneal tendon tear, osteochondral defect, nor syndesmotic injury that would lead one to order an MRI scan to guide treatment. The use of a CAM boot and nonweightbearing is not appropriate for the treatment of an ankle sprain in this setting. If the patient was unable to bear weight or an occult fracture was suspected, then non-weightbearing followed by an MRI can be considered. Primary repair of the lateral collateral ligaments is not indicated even in high-level athletes at this time in the setting of an isolated lateral low ankle sprain, regardless of the grade of injury. Although there is some new discussion that it may be beneficial in high-level athletes, there is not sufficient evidence to justify this treatment method at this time. Therefore, the most appropriate answer is functional bracing with a laceup ankle brace with concomitant physical therapy for peroneal strengthening and proprioception. If the patient develops persistent instability despite this protocol, then a secondary reconstruction of the lateral collateral ligaments is considered.
Question 65 of 100
Figure 1 shows the deformity that developed in an active 49-year-old woman who had previously undergone a bunion correction. The patient's great toe is easily corrected to a neutral position but tends to spring back to a varus position. She reports pain in the first metatarsophalangeal joint and has difficulty wearing most shoes. What is the most appropriate management plan?
A. Split extensor hallucis longus tendon transfer
B. Great toe fusion
C. Medial soft-tissue release and lateral capsule plication
D. Metatarsal osteotomy, medial capsule release, and split extensor hallucis longus tendon transfer1
R: D
Osteotomy and tendon transfer is the management of choice. The previous bunion correction resulted in excessive translation of the metatarsal head. The orthopaedic surgeon must first correct the bony deformity and allow the proximal phalanx to sit in a congruent position. The next step is to reconstruct the soft-tissue components and this can be done by releasing the medial capsule, and transferring part of the extensor hallucis longus tendon into the proximal phalanx, under the intermetatarsal ligament laterally. All three procedures are needed to adequately correct this deformity. A great toe fusion is indicated for an uncorrectable deformity or in an older patient.
Question 66 of 100
Operative arthroscopy is performed in a 25-year-old man with ankle impingement caused by Bassett's ligament. Appropriate treatment involves resecting the thickened distal fascicle of which ligament?
A. Anterior tibiotalar
B. Anterior inferior tibiofibular
C. Transverse tibiofibular
D. Posterior inferior tibiofibular
R: B
Thickening of Bassett's ligament (distal fascicle of the anterior inferior tibiofibular ligament) represents a common cause of anterolateral ankle impingement and is appropriately treated with arthroscopic resection. The anterior tibiotalar ligament may also cause anterior soft-tissue impingement, while the transverse tibiofibular and posterior inferior tibiofibular ligaments are located in the posterior ankle.
Question 67 of 100
A patient falls off a roof and sustains the fracture shown in Figure 1. What is the most likely complication that results from injury to the structure that is located at the arrow?
A. Paresthesias on the plantar aspect of the foot
B. Pain or popping with great toe flexion
C. Loss of the arch
D. Inability to flex the lesser toes
R: B
The arrow points to the sustentaculum tali, which is fractured off the tuberosity of the calcaneus. The flexor hallucis longus (FHL) tendon runs directly under this structure. An injury to this structure could cause stenosis around the FHL tendon which would cause pain with great toe flexion. Paresthesias on the plantar aspect of the foot refers to the medial plantar nerve. Loss of the arch refers to the plantar fascia which attaches at the calcaneal tuberosity. Inability to flex the lesser toes refers to the flexor digitorum longus tendon which runs superior to the sustentaculum.
Question 68 of 100
A 45-year-old man with a history of type 1 diabetes is seen in clinic with 5 months of foot pain. He does not recall any trauma or inciting event. He is casted initially and able to return to work with a CROW boot. He is seen 1 year later with new foot swelling and the radiographs taken at that time are shown in Figures 1 through 3. Which class of drugs has been shown to help improve pain and prevent bone resorption in these patients?
A. Nonsteroidal anti-inflammatory drugs
B. Antidepressants
C. Bisphosphonates
D. Disease-modifying anti-rheumatic drugs
R: C
Bisphosphonates have shown good short-term results in reducing bone resorption. Early studies with the drugs showed improvement in pain scores and improvements in bone turnover markers.
Question 69 of 100
A 45-year-old woman with type 2 diabetes (BMI 38, Hgb A1c 7.4) has a grade II ulcer under the first metatarsal head. Previous treatment with a custom orthosis and total contact casting has provided only temporary healing. Her ankle-brachial index is 0.95, she has no foot deformity, and there is no evidence of infection. What is the next most appropriate step in management?
A. Gastrocsoleus recession and peroneus longus to brevis tendon transfer
B. Resection of the first metatarsal head
C. First-ray amputation
D. Transmetatarsal amputation
R: A
Ulceration in neuropathy is a consequence of increased pressure. When it occurs in the forefoot, it is frequently associated with a tight heel cord. In addition, lesions under the first metatarsal often have an associated overpull of the peronus longus, plantar flexing the first metatarsal. Before bony resections are contemplated, a fractional lengthening of the heel cord (gastrocsoleus recession) as well as a peroneal longus to brevis tendon transfer will decrease the forefoot pressures and leads to decreased ulcer recurrence rates.
Question 70 of 100
Figures 1 through 3 are the radiographs of a 21-year-old Division 1 collegiate male track athlete who presents late season with a 3-week history of pain localized to the lateral midfoot. He denies any acute injury. Pain was initially mild; however, over the past 3 days, he can no longer bear weight. He undergoes successful surgical treatment with 4-month follow-up radiographs shown in Figures 4 through 6. What represents the greatest risk factor for refracture in this athlete?
A. Treatment with cannulated intramedullary screw
B. Varus hindfoot alignment
C. Early return to running after radiographic union
D. Inadequate dietary intake of vitamin D
R: B
Fifth metatarsal stress fractures are one of the most common injuries seen in running athletes. Jones fractures, occurring in Zone II, have a 25% risk of nonunion. Treatment can either be nonsurgical with nonweightbearing cast immobilization or surgical with intramedullary screw fixation. Surgical management is favored in athletes because of a higher rate of union and lower risk of refracture. Cavovarus foot alignment is a significant risk factor for these injuries, as well as for nonunion and refracture after successful treatment of these injuries. Vitamin D deficiency is a risk factor for nonunion but is unlikely in a 21-year-old male athlete. Assuming radiographic union was achieved, early return to running is not a risk factor for refracture.
Question 71 of 100
Figures 1 through 3 are the radiographs of a 35-year-old woman who has had 7 years of progressive ankle pain. She experiences stiffness and pain despite the use of an ankle-foot orthosis. Examination reveals pain along the anterior tibiotalar joint without tenderness to the subtalar or talonavicular joints. What is the most appropriate surgical intervention?
A. Total ankle arthroplasty (TAA)
B. Ankle arthrodesis
C. Tibiotalocalcaneal arthrodesis
D. Anterior tibial exostectomy with Achilles lengthening
R: B
This patient has isolated posttraumatic ankle arthritis with significantly decreased ankle range of motion that is best treated with an isolated ankle arthrodesis to eliminate pain. Because this patient is younger than 50 years of age, she is not a candidate for ankle arthroplasty. Additionally, TAA outcomes among patients who have a posttraumatic etiology are worse than for those with osteoarthritis. Tibiotalocalcaneal arthrodesis is suited for patients with associated subtalar arthritis or rigid hindfoot deformity that necessitates correction. Anterior tibial exostectomy will not resolve the underlying arthritic pain and is not indicated.
Question 72 of 100
A patient with pes planus has the primary complaint of pain and deformity of the second toe. The radiograph shown in Figure 1 reveals the deformity of the second toe. In addition to a metatarsal osteotomy, what is the recommended treatment for this deformity?
A. Resection of the metatarsal head
B. Arthrodesis of the metatarsal phalangeal joint
C. Flexor to extensor tendon transfer
D. Flexor tendon release
R: C
The radiograph shows a complete dislocation of the second metatarsophalangeal (MTP) joint. Instability of the lesser MTP joints is classified in five grades. Grade 0 is a normal stable joint, Grade 1 translates 50% on stress examination, Grade 2 translates 100%, Grade 3 dislocates but is reducible, and Grade 4 is fixed dislocated joint. The radiograph shows a Grade 4 lesion that is dislocated and fixed. Recent studies have popularized the repair of the plantar plate, but Grade 4 lesions typically do not have sufficient remaining plate for a direct repair and the flexor-to-extensor tendon transfer is recommended. A shortening osteotomy with or without a tendon transfer has also been recommended for the dislocated MTP joint.
Question 73 of 100
Figures 1 through 3 are the AP and lateral radiographs of a 55-year-old man who has a painful first metatarsophalangeal (MTP) joint. What treatment has the most reliable long-term results?
A. Implant resurfacing arthroplasty
B. MTP arthrodesis
C. Cheilectomy
D. Interposition graft arthroplasty with platelet-rich plasma
R: B
Arthrodesis remains the benchmark standard surgical option for end-stage arthritis of the great toe MTP joint, regardless of age or activity level. Cheilectomy for dorsal osteophyte resection can improve shoe wear limitation but will not reliably improve pain or increase joint motion. Joint arthroplasty retains some joint motion, but at the expense of metatarsal shortening, which may cause transfer metatarsalgia and impose problematic salvage fusion because of bone loss, if unsuccessful. Synthetic or any optional cartilage substitute appears to be favorable, with pain relief and retained motion, but no long-term outcomes have been reported. Lastly joint arthroplasty with interposition tendon graft yields pain relief with retention of some motion, but robust studies have not been reported.
Question 74 of 100
A 45-year-old man has right ankle pain after a skateboarding injury. He is able to weight bear, but has pain when standing and walking. On examination, the patient has tenderness medially near the medial ankle joint line. He has tenderness proximally over the fibula as well. Nonstanding radiographs of the ankle show no fractures. Full-length tibiofibular radiographs show a proximal fibula fracture. What is the best next step in management?
A. Stress radiographs of the ankle
B. CT scan of the ankle
C. MRI scan of the ankle
D. Cast immobilization
R: A
In any rotational ankle injury, a proximal fibular fracture should be suspected. In this case, the patient has tenderness proximally. This raises the concern of a syndesmotic injury or a Maisonneuve injury. In evaluating the syndesmosis and its stability, the first step, if able to tolerate, should include weight-bearing stress radiographs.
Question 75 of 100
A 28-year-old construction worker with a body mass index (BMI) of 31 sustained a Weber C fracture 3 years ago. An open reduction and internal fixation was performed, but he developed degenerative changes in the ankle as seen in Figure 1. Management consisting of bracing, shoe modifications, and other modalities has failed to provide relief. He is symptomatic enough that he wants definitive treatment. What is the best treatment option at this time?
A. Bipolar allograft replacement of the tibial plafond and talar dome
B. Low profile total ankle arthroplasty
C. Arthroscopic ankle débridement
D. Ankle fusion
R: D
Ankle fusion will provide the most reliable pain relief and function for this young manual laborer. At his young age and with a BMI of 31, both total joint arthroplasty and allograft replacement are controversial. The radiographs show degenerative changes that are too far advanced for an arthroscopic ankle debridement to be of any benefit.
Question 76 of 100
Figure 1 is the MRI scan of an active 63-year-old golfer who fell getting out of a sand trap. He describes a popping sensation and difficulty with ambulation. On examination, he has decreased resting tension of the tendon and has no plantarflexion with Thompson's testing. What percentage of these injuries are diagnosed >4 weeks out from the injury?
A. <10 %
B. 10% to 25%
C. 30% to 50%
D. >50%
R: B
Oftentimes, acute Achilles tendon ruptures can initially be missed. Risk factors for delayed diagnosis include high BMI, injury during nonsporting activity, and age >55 years. When presenting in a delayed fashion, there is often not significant pain. Symptoms may be somewhat vague and related to difficulty with gait or stairs rather than pain.
Question 77 of 100
A 44-year-old woman with forefoot pain has pain with weight bearing during toe-off. She reports the pain is worse when she is barefoot and better when wearing tennis shoes. She has no numbness or tingling. Examination reveals increased pain with second toe dorsiflexion and plantar flexion. Traction to the second toe decreases pain with motion. She has no pain with medial lateral forefoot compression. Radiographically, her second metatarsal is longer than the first. What is the most likely diagnosis?
A. Second metatarsal stress fracture
B. Second metatarsophalangeal (MTP) synovitis
C. Second to third web space neuroma
D. Transfer metatarsalgia
R: B
A test for metatarsophalangeal synovitis is pain with motion that is decreased with traction and motion. A patient with a neuroma typically has less pain when barefoot, does not have pain with MTP motion, and often has pain and a click with medial lateral forefoot compression. A metatarsal stress fracture would cause pain with weight bearing. The drawer test is the most useful test for diagnosis of instability of the metatarsophalangeal joint. Transfer metatarsalgia could be considered but typically does not cause pain with toe motion or have a positive drawer test.
Question 78 of 100
Figures 1 and 2 are the radiographs of an active 65-year-old woman who has a 3-year history of increasing foot pain and flattening of the left foot. Inversion strength is 5+ and does not reproduce her symptoms. Bracing and nonsteroidal anti-inflammatory drugs have failed to provide adequate relief. She has a supple hindfoot and normal heel cord flexibility. What is the most appropriate treatment?
A. Talonavicular arthrodesis
B. Medial Lisfranc arthrodesis
C. Subtalar arthrodesis
D. Triple arthrodesis
R: B
The patient has degenerative changes with collapse at the tarsometatarsal joints; therefore, a fusion of the Lisfranc joint is indicated. The other responses are hindfoot procedures that will not address the tarsometatarsal joint.
Question 79 of 100
A 36-year-old man sustains an eversion and external rotation ankle injury, and physical examination demonstrates a positive squeeze test. What is the next most appropriate step in management?
A. Supine radiographs of the ankle
B. MRI scan of the ankle
C. Walker boot
D. Full-length tibiofibular radiographs
R: D
Radiographic assessment is the most appropriate next step in the diagnostic management of suspected ankle syndesmotic sprain injury and includes both standing (or simulated weight-bearing) radiographs of the ankle, as well as full-length tibiofibular radiographs. External rotation stress radiographs of the ankle and MRI scan are adjunct imaging procedures, and walker boot represents a treatment option for stable low-grade syndesmotic sprains.
Question 80 of 100
A patient undergoes 6 weeks of intravenous antibiotics for osteomyelitis after the transmetatarsal amputation of the third ray for osteomyelitis. The wounds from the surgery heal without complication. Erythema develops days after discontinuation of the antibiotics. MRI scans of the second toe are shown in Figures 1 and 2. Radiographs of the second ray show no bone destruction. Appropriate treatment and its justification would be
A. amputation of the second ray is required to treat the infection.
B. transmetatarsal amputation of the second ray may reduce the duration of antibiotic treatment.
C. observation since MRI findings are not related to the recurrence of the erythema.
D. surgical biopsy required to determine if the MRI findings are due to infection.
R: B
The MRI changes noted in the second metatarsal are consistent with osteomyelitis and no abscesses are noted. With recurrence of the clinical signs of infection, it is reasonable to infer the changes seen on MRI scan are from osteomyelitis. A bone biopsy would be useful to determine the appropriate antibiotics, especially in a case where there has been failure of medical treatment. It has been shown that resection of infected bone reduces the bioburden of the infection and can reduce the duration of antibiotic treatment. For a patient without signs of bone destruction, antibiotic therapy without surgery is considered a reasonable treatment option.
Question 81 of 100
A 25-year-old student has foot pain and swelling after being tackled while playing football. He was unable to continue playing that day and non–weight-bearing radiographs were reported to be normal. The examination shows moderate swelling and diffuse tenderness at the forefoot. Plantar ecchymosis is noted in the midfoot. What is the best next step?
A. Six weeks of protected weight bearing in a boot
B. Weight-bearing radiographs of both feet
C. Referral to physical therapy
D. Continued crutches and repeat evaluation in 2 weeks
R: B
The patient has a history and examination concerning for an injury to the tarsometatarsal, or Lisfranc, joints, especially given the finding of plantar ecchymosis. Although the radiographs are unremarkable, they are also non-weight bearing and may not show instability. For this reason, additional imaging is indicated. Weightbearing radiographs can demonstrate instability of the Lisfranc joint. MRI would be an appropriate test if weight-bearing radiographs do not demonstrate instability. Protected weight bearing or physical therapy is not indicated because a midfoot injury has not been ruled out. Repeating a clinical evaluation in 2 weeks may only delay diagnosis.
Question 82 of 100
A 63-year-old man sustained the injury shown in the radiographs in Figures 1 and 2. What complication can occur if the displaced fracture fragment is malreduced?
A. Flexor hallucis longus tendon entrapment
B. Calcaneofibular ligament laxity
C. Achilles tendon impingement due to Haglund deformity D.Subtalar arthritis
R: C
In tongue-type calcaneus fractures as seen in these images, malreduction of the fracture fragment can cause the development of a bony prominence similar to a Haglund deformity, which can impinge on the Achilles tendon.
Question 83 of 100
Figure 1 is the lateral radiograph of a 40-year-old laborer who sustained a displaced intra-articular calcaneus fracture that was treated nonsurgically 1 year ago. He now reports pain with ankle dorsiflexion, as well as subfibular impingement. What is the most appropriate surgical treatment?
A. Lateral wall ostectomy
B. In situ subtalar arthrodesis
C. Distraction subtalar arthrodesis with lateral wall ostectomy D. Ankle and subtalar arthrodesis
R: C
The patient has a calcaneal fracture malunion, with symptomatic subtalar arthritis and anterior ankle and lateral subfibular impingement. Distraction subtalar arthrodesis addresses subtalar arthritis and anterior impingement and lateral wall ostectomy relieves symptoms of lateral impingement. The other procedures do not address all facets of the patient's symptoms.
Question 84 of 100
Figure 1 is the radiograph of a 33-year-old runner who recently decided to begin running barefoot on trails. Since his transition to running without shoes 3 months ago, he has been having pain in the second metatarsophalangeal (MTP) joint. He feels like he is walking on a stone, notes edema in the ball of his foot, and has started to see a deviation of the second toe. What is the most likely etiology of these symptoms and findings?
A. Tear of the plantar plate
B. Second MTP joint synovitis
C. Second metatarsal stress fracture
D. Flexor tendonitis
R: A
Lesser-toe plantar plate injuries are becoming increasingly recognized. Patients typically have an increase in pain, a positive Lachman test result upon examination, and deviation of the MTP joint. On radiograph, MTP subluxation can be appreciated. Nonsurgical treatment with a metatarsal pad may be attempted. Many patients who have surgery will have a partial or full tear of the plantar plate. The repair necessitates reinsertion of the plantar plate to the base of the proximal phalanx.
Question 85 of 100
One of the reported disadvantages of ankle arthrodesis is progression of degenerative joint disease on radiographs at >5-year follow-up. A reported advantage of total ankle replacement (TAR) is that during long-term follow-up, the adjacent joints demonstrate
A. improvement in the radiographic signs of degeneration.
B. no change in the radiographic signs of degeneration.
C. less progression in the radiographic signs of degeneration.
D. fewer arthrodesis due to progression of degeneration.
R: C
A reported advantage of TAR is a reduction of adjacent subtalar and talonavicular degenerative joint disease on radiographs. The maintenance of normal tibiotalar motion is suggested to reduce the stress on the adjacent joints. However, the actual clinical implications of the noted reduced degeneration on radiographs has not been documented. Although it is speculated that this reduction in stress on adjacent joints will lead to fewer adjacent joint arthrodesis, this outcome has not been reported in studies comparing TAR with ankle arthrodesis.
Question 86 of 100
Figures 1 and 2 are the radiographs of a 34-year-old woman who has a painful ankle following an attempted fusion of her ankle 6 months ago. Infection work-up was negative. The subtalar joint is pain free with manipulation. What is the most appropriate treatment?
A. Removal of hardware and an intramedullary rod tibiocalcaneal fusion
B. Use of an external bone stimulator
C. Removal of the plate and screws and the addition of an internal bone stimulator D. Revision fusion with stable compression fixation and bone graft
R: D
Because the subtalar joint looks normal, and there is enough talus to work with, the subtalar joint should be spared and only an ankle fusion performed, especially in view of the patient's young age. There is clearly a distraction at the fusion site and the distal fixation is loose. The patient needs a formal revision with a transfibular approach with compression screws. Removal of hardware and/or a bone stimulator will not be sufficient.
Question 87 of 100
Figures 1 and 2 are the radiographs of a 20-year-old collegiate varsity athlete who reports lateral foot pain. What is the most appropriate management at this time?
A. Orthosis and non-weight-bearing status
B. Orthosis, weight bearing as tolerated, and use of a bone stimulator
C. Short-leg cast
D. Internal fixation
R: D
Fractures of the fifth metatarsal proximal metaphyseal-diaphyseal junction generally occur in young athletic patients and have relatively high rates of delayed union or nonunion with nonsurgical management. The fracture occurs in the hypovascular zone between the insertion of the peroneus brevis and tertius. These tendons cause a shearing across the fracture site, preventing stability and healing. Nonsurgical functional bracing or casting may lead to a high rate of delayed union and nonunion. Internal fixation in the high-level athlete leads to the most predictable healing of the fracture in a timely fashion. The use of bone stimulators for this fracture is controversial.
Question 88 of 100
Figures 1 through 3 are the standing and stress radiographs of the left ankle of a 48-year-old man with history of chronic left ankle instability who is scheduled for a lateral ligamentous repair procedure.
Which anatomic factor may predispose this patient to recurrent injury in the postsurgical setting?
A. Posterior tibial tendon dysfunction
B. Varus hindfoot
C. Anterior ankle impingement
D. Metatarsus primus elevates
R: B
The standing radiographs show a foot with cavovarus deformity. Varus deformity of the hindfoot predisposes the patient to ankle inversion injury and is an anatomic risk factor for recurrent instability following operative treatment. Posterior tibial tendon dysfunction is associated with pes planovalgus deformity, and metatarsus primus elevatus describes dorsal deviation of the first metatarsal. Anterior ankle impingement is not associated with recurrent ankle instability.
Question 89 of 100
A 50-year-old woman with a mild flexible planovalgus foot deformity has lateral hindfoot pain. What is the simplest modification of her shoe wear to help offload the lateral hindfoot?
A. Medial hindfoot posting B. Lateral hindfoot posting
C. Semi-rigid foot orthotic
D. Accommodative foot orthotic
R: A
Lateral hindfoot pain is often the result of impingement as the planovalgus foot pronates and abducts. Medial hindfoot posting or wedge will elevate the medial aspect of the heel and decrease the lateral hindfoot pressures. This may be done in a dress shoe as well as a lace-up shoe with a simple heel posting wedge. Adding lateral hindfoot posting would increase the lateral pressures. An orthotic, whether semi-rigid or accommodative, will support the arch but without medial hindfoot posting, the foot will often still pronate and abduct with continued lateral hindfoot pressure.
Question 90 of 100
A 19-old-college basketball player sustains the fracture shown in Figure 1. This fracture is commonly associated with what foot deformity?
A. Pes planovalgus
B. Pes cavovarus
C. Hallux valgus
D. Midfoot abduction
R: B
The patient sustained a zone II proximal fifth metatarsal. This fracture is commonly associated with hindfoot varus and pes cavus. This foot deformity also increases the risk of nonunion and the risk of refracture.
Question 91 of 100
Figure 1 is the lateral radiograph of a 55-year-old man who is evaluated for a 2-year history of pain and stiffness of his right metatarsophalangeal (MTP) joint. Upon examination he has dorsal bossing, severe crepitation, and pain with passive range of motion. There is pain with the 'grind' test. Dorsiflexion is limited to 0 degrees. No sesamoid tenderness is present. What is the most appropriate surgical treatment?
A. Chevron bunionectomy
B. Cheilectomy and removal of loose body
C. MTP arthrodesis
D. Resection of the proximal phalanx
R: C
The radiograph reveals end-stage degenerative changes of the first MTP joint with a dorsal loose body. MTP arthritis and decreased joint dorsiflexion is referred to as hallux rigidus. A chevron bunionectomy is used to correct hallux valgus deformity without arthritis. The cheilectomy is used in lesser degrees of joint destruction. Resection of the proximal phalanx results in a floppy toe and is generally not recommended.
Question 92 of 100
A 35-year-old man has had pain along the posteromedial ankle and hindfoot for 6 months. The pain is present with prolonged walking and standing and severely limits his ability to run. Examination reveals an asymmetric alignment with increased hindfoot valgus and forefoot abduction of the affected lower extremity. Swelling and tenderness is noted along the course of the affected tendon. He is able to perform a double limb heel rise with inability to perform a single limb heel rise on the affected leg. A gastrocnemius contracture is noted. What is the most appropriate initial treatment?
A. A half-inch heel lift and physical therapy for eccentric strengthening of the affected tendon
B. Custom molded hinged ankle-foot orthosis (AFO) and physical therapy to strengthen the affected tendon
C. Gastrocnemius recession followed by custom-molded orthotics D. Spring ligament reconstruction and gastrocnemius recession
R: B
The patient is presenting with posterior tibial tendon dysfunction (PTTD) with an associated gastrocnemius contracture. The most likely diagnosis is stage 2 PTTD given the pain along the medial hindfoot over the posterior tibial tendon (PTT) and inability to do a single limb heel rise. The examination finding would have a supple hindfoot, and one important aspect to note when considering surgical reconstruction is whether residual forefoot supination is present after hindfoot correction. If supination is present, then the medial column must be plantarflexed through either a Cotton osteotomy/naviculocuneiform fusion if grossly unstable/first tarsometatarsal fusion if grossly unstable.
In this clinical scenario, the patient has not had any treatment, and therefore, nonsurgical management should be considered. The use of a half-inch heel lift does accommodate the contracture; however, eccentric strengthening is ideal for Achilles tendinosis. A gastrocnemius recession would surgically correct the contracture, followed by an orthotic, if ultimately required; however, as there are no data to determine which patient will respond to nonoperative intervention, a gastrocnemius recession is not indicated at this time. A spring ligament reconstruction and gastrocnemius recession would be appropriate in a patient with a traumatic acute spring ligament rupture. Those patients may have deformity as in this case; however, they are typically able to perform a single limb heel rise, despite the pain as the PTT is functioning normally. In this case, the patient has no trauma and has an inability to perform a single limb heel rise, in addition to pain over the PTT, more consistent with stage II PTTD.
Given the clinical scenario, the most reasonable treatment is the initiation of the nonoperative protocol for flexible PTTD. This treatment includes the use of a custom-molded hinged AFO and focused physical therapy to strengthen the tendon. Although, this can be cumbersome, it is the most effective method to minimize the need for surgery. Successful resolution of pain after this treatment regimen, which can require up to one year of treatment, can be maintained if shoe orthotics are utilized to maintain the pain relief.
Question 93 of 100
Figures 1 through 3 are the lateral radiograph and MRI scans of a 32-year-old woman who reports a 3-week history of heel pain, tenderness, swelling, and onset following an increase in running activity. What is the most likely diagnosis?
A. Plantar fasciitis
B. Atrophic heel pad
C. Retrocalcaneal bursitis
D. Stress fracture of the calcaneus
R: D
The sagittal T1-weighted MRI scan reveals a linear streak of low signal intensity consistent with a stress fracture of the posterior calcaneal tuberosity. The surrounding area of hypointensity on the T1 MRI scan and the hyperintensity on the T2 MRI scan represent bone contusion, hemorrhage, and edema within the calcaneus, with an unremarkable radiograph. The plantar fascia, retrocalcaneal bursa, and heel pad all display normal signal in the accompanying MRI scans.
Question 94 of 100
A 19-year-old woman sustained a displaced talar neck fracture while cliff jumping. The fracture is managed with open reduction and internal fixation. Which of the following best describes the findings in the 2-months postoperative radiographs shown in Figures 1 and 2, and subsequent treatment plan?
A. There is a positive Hawkins sign, indicating the patient is unlikely to develop osteonecrosis.
B. There is a positive Hawkins sign, indicating the patient has developed osteonecrosis.
C. No Hawkins sign is visible, and therefore the patient is not likely to develop osteonecrosis.
D. No Hawkins sign is visible; therefore, the patient should be kept non-weight-bearing until a Hawkins sign appears
R: A
The radiographs reveal a positive Hawkins sign, a subchondral lucency in the talar dome best seen on a mortise radiograph indicating viability of the talar body. Once a Hawkins sign appears, it is unlikely that that the patient will develop osteonecrosis. Osteonecrosis is best diagnosed with radiographs. A Hawkins sign typically will appear at 6 to 8 weeks after fracture; however, the absence of a Hawkins sign at that time does not necessarily indicate osteonecrosis. Most authors agree that even in the absence of a Hawkins sign, weight bearing can commence at 10 to 12 weeks after surgery.
Question 95 of 100
Figure 1 is the radiograph of a 52-year-old patient with painful lesser toe deformities. Examination of the second digit shows hyperextension deformity at the metatarsophalangeal (MTP) and flexion of the distal interphalangeal and proximal interphalangeal joints. The base of the second proximal phalanx has excessive excursion with a dorsal stress. Callousing is noted beneath the second metatarsal head. Which surgical reconstruction technique will best re-establish a plantigrade position of the toe?
A. Percutaneous Achilles tendon lengthening
B. Extensor digitorum longus (EDL) to flexor hallucis brevis tendon transfer
C. Dorsiflexion metatarsal osteotomy
D. Plantar plate repair
R: D
The patient presents with an advanced claw toe deformity, including instability at the MTP joint. The radiograph shows a lateral deviation of the proximal phalanx that is consistent with injury to the plantar plate. This finding indicates loss of integrity of the plantar plate. Of the surgical options, only plantar plate repair improves the extension deformity at the MTP joint, allowing the toe to assume a plantigrade position. The patient exhibits a tight Achilles tendon. Both Achilles tendon lengthening and a dorsiflexion metatarsal osteotomy may be indicated for metatarsalgia, but it plays no role in correcting the lesser toe deformity. An EDL transfer would exacerbate the deformity.
Question 96 of 100
A tall, thin 17-year-old basketball player and his parents request an evaluation of his flexible (hypermobile) pes planus/planovalgus foot deformities. As part of his evaluation, the orthopaedic surgeon notes pectus excavatum, disproportionately long arms, and scoliosis. In addition to providing treatment of his feet, what test or evaluation should the patient be referred for?
A. Cardiovascular evaluation
B. MRI of the spine
C. Radiographs of the hip
D. Genetic testing
R: A
The current diagnostic criteria for Marfan syndrome, called the Ghent criteria, are based on clinical findings and family history. The role of genetic testing in establishing the diagnosis is limited, because testing for FBN1 mutations is neither sensitive nor specific for Marfan syndrome. By making the diagnosis and arranging for cardiovascular evaluation, the orthopaedic surgeon can help prevent sudden death in these patients. The cardiovascular manifestations, including dissection and dilation of the ascending aorta and mitral valve prolapse, are responsible for nearly all of the precocious deaths of patients with Marfan syndrome. Patients with Marfan syndrome do have problems with protrusio acetabuli and scoliosis but the life-threatening problem that must be considered is the risk of cardiovascular sudden death.
Question 97 of 100
Figure 1 shows the radiographs of a 45-year-old woman with pain over the lateral aspect of the fifth metatarsal head. She has tried shoes with a wider toebox, pads, and spacers without relief. What technique would lead to the highest rate of failure?
A. Minimally invasive distal metatarsal osteotomy
B. Distal chevron metatarsal osteotomy
C. Weil (long oblique) metatarsal osteotomy
D. Simple resection of the fifth metatarsal head eminence
R: D
Many bunionette deformities can be managed nonsurgically. Shoe wear modification, silicone sleeves, and pads can all be utilized. When this fails, surgery can be indicated. Type I deformities are simply an enlarged metatarsal head or boney exostosis. In this case, simple resection with capsular tightening is adequate. Type II deformities are a congenital bowing of the fifth metatarsal. Type II indicates a true increase in the normal intermetatarsal (IM) angle (normally 6-8°). In patients with an increase in their 4-5 IM angles, osteotomies are preferred for treatment. Simple bunionette shaving does not correct the deformity and continued symptoms are likely. In general, distal osteotomies are best for smaller IM angles (<12°), and more proximal osteotomies are required for deformities with larger IM angles. This patient has an increased IM angle that would be best served by a metatarsal osteotomy.
Question 98 of 100
Figures 1 and 2 are the AP and lateral radiographs of a 48-year-old woman who underwent a revision Lapidus procedure for left hallux valgus and now has pain under the second metatarsal head with callus that is refractory to nonsurgical treatment. What is the most appropriate surgical treatment for this patient's symptoms?
A. Chevron osteotomy of the first metatarsal
B. Shortening osteotomy of the second metatarsal
C. Dorsiflexion osteotomy of the first proximal phalanx
D. Medial closing wedge osteotomy of the first proximal phalanx
R: B
Reduction in the first ray length after surgical correction of hallux valgus can lead to the development of transfer lesions in the lesser metatarsal heads. The most appropriate surgical treatment for transfer metatarsalgia with a relatively elongated second ray is shortening osteotomy of the second metatarsal. Chevron osteotomy of the distal first metatarsal is used for correcting hallux valgus, whereas osteotomies of the first proximal phalanx are used as adjunct procedures for treating hallux valgus interphalangeus and hallux rigidus.
Question 99 of 100
A 25-year-old man who sustained a Weber B fibular fracture 2 years prior, undergoes open reduction and internal fixation with uneventful healing. He is currently experiencing pain along the anterior aspect of the ankle that occurs when going up and down stairs, and during lunges and running. He has no pain with daily activities, biking, or swimming. Examination reveals mild pain with palpation along the anterior ankle that is increased with forced dorsiflexion. Range of motion is 5° of dorsiflexion that does not improve with knee flexion, and 40° of plantar flexion. Three views of the ankle weight bearing are shown in Figures 1 through 3. What is the most appropriate surgical management to alleviate his pain?
A. Distraction arthroplasty with hinged external fixator
B. Achilles lengthening
C. Anterior closing wedge osteotomy tibia
D. Arthroscopic ankle debridement and cheilectomy
R: D
The patient has a well-healed distal fibular fracture with a well-maintained mortise without any evidence of joint space narrowing on radiographs. His primary complaint relates to impingement along the anterior ankle with dorsiflexion activity. In contrast to posttraumatic arthritis, he has no pain with normal weight-bearing activity. The radiographs reveal an anterior distal tibial spur and, in conjunction with his clinical complaints, this is consistent with anterior ankle impingement. This is commonly seen in patients with recurrent ankle instability or low energy trauma, in which the articular cartilage is still maintained; however, the injury has led to distal tibial spur formation. A secondary spur may be noted off of the talar neck, most commonly medially. The tibial spur is most commonly noted laterally, although it can extend across the entire ankle joint. The most effective method with the lowest morbidity is an arthroscopic ankle debridement and cheilectomy. An Achilles tendon lengthening in isolation would not be appropriate, as it will not resolve the bony block to motion nor improve his pain. Following the cheilectomy, one can consider an intraoperative assessment of motion and when indicated, a gastrocnemius recession or Achilles tendon lengthening could be performed. An anterior closing wedge osteotomy would achieve an improvement in the functional dorsiflexion; however, this is a more difficult procedure with increased risk and without any evidence of malunion, with a clear distal tibial spur, an osteotomy is not the most appropriate procedure. Distraction arthroplasty has no role here, as the patient does not have ankle arthritis.
Question 100 of 100
Figures 1 through 3 are the radiographs of a 10-year-old boy who has a 6-month history of progressive heel pain. The patient is a year-round soccer player and now experiences pain with most every step. What is the most appropriate management?
A. MRI
B. Activity modification
C. Calcaneal epiphysiodesis
D. Percutaneous Achilles tendon lengthening
R: B
The patient has calcaneal apophysitis, an overuse syndrome common in children ages 9 to 12 years. Symptoms are usually the result of excess tension and a tight heel cord. Management includes activity modification, as well as heel cord stretching, nonsteroidal anti-inflammatory drugs, icing, and other modalities. Radiographs are typically negative; MRI is unnecessary. The condition is self-limiting, in that the symptoms fully resolve once the apophyses fuses, such that surgery is rarely indicated.